You are on page 1of 60

SHIKHAR

SUNTAR
IT-JEE
XENON ENTRANCE PREPARATION BOO
READ THE FOLLOWING THEOREMS :
LINES AND ANGLES
Theorem 1 : a+p=x
The adjacent angles which one straight line makes with another straight
line on one side of it are together equal to two right angles. (Supplementary
Angles)
Theorem 2 :
If two straight lines cut one another, the vertically opposite angles B B
are equal.
Theorem 3 :
If a straight line cuts two parallel lines, it makes
(i) the alternate angles equal to one another;
(i) the exterior angle equal to the interior opposite angle on the same side of the cutting line;
(iii) the two interior angles on the same side together equal to two right angles.

AT
L3 =15, L4=16, L1 =13,
£L2=L4, £8=1L6, LT =145
Z4 + £5 =2 Right Angles
£3 + £6 =2 Right Angles
Theorem 4 :
If a straight line cuts two other straight lines so as to make
(i) the alternate angles equal, or
(i) and exterior angle equal to the interior opposite angles on the same side of the cutting line, or
(iii) the interior angles on the same side equal to two right angles;
then in each case the two straight lines are parallel.
If
L3 =45,/44= 26, L1 =3,
L2=L4, L8=16,LT=L5
Z4 + £5 = 2 Right Angles
43 + £6 = 2 Right Angles

=414
Theorem 5 :
If there are three or more parallel straight lines and the intercepts made
by them on any transversal are equal, then the corresponding intercept on
any other transversal are also equal.
If PQ=QR
= ST =TU
PROBLEM SET-1__ (3 Marks each, -1 for wrong answer) *
1. MN and OP are parallel to each other and EF is the transversal. ZFBO = (3x + 5)° and ZMAE = (2x)°.
The supplement of ZPBA : complement of /NAB
E'

(A)2:7 (B) 7:2 (C) 1:1 (D) 1:6


SHIKHAR : Indra Complex Vijaya Nagar, Chetakpuri Gw: r 0751-3551081
SHIKHAR
SUNTAR
IT-JEE
2.

x+50!
y-20°

In the above given figure, ¢ | m and x and y are complementary angles. Find x and y.
(A) 50°, 40° (B) 20°, 70° (C) 10°, 80° (D) 30°, 60°
3. MN and OP are two straight lines intersecting at R. If ZORN = 5/2 (/ORM), find ZNRP : /MRP.
M P

R
o N
(A)2:5 B)5:2 ©1:1 D3:7
4. Inthe below figure PQ || RS and PR | QS. Each pair of parallel rays is intersecting the other pair. ZA,
4B, «C and £D, are the angles formed. If «C = 110°, then 4B =

(A) 90° (B) 110° (C) 100° (D) 120°


5. Inthe figure shown, PQ | RS and SM | TN. Then measure of angle o is

B T Q
(A) 58° (B)118° (C)89° (D)91°
Subjective Type (4 Marks each, -1 for wrong answer)
6. In the given figure if ZAOB = 132°, ZCOD = 141°, if measure of ZBOD equals x°, then x is
B

A o) C
PROBLEM SET-2 (3 Marks each, —1 for wrong answer) *

2. InAPQR,if /R > 2Q, then


«(A) QR > PR (B) PQ > PR (C) PQ PR (D) QR < PR

3.

In the figure above, ABE is a triangle and AC = AD. What is the measure of ZCBA?
(A) 30° (B) 40° (C) 60° (D) 80°
4. InaAABC, AB = AC and AD L BC, then
(A) AB<AD (B) AB > AD (C) AB = AD (4) AB< AD

SHIKHAR : Indra Complex Vijaya Nagar, Chetakpuri Gwalior 0751-3551081


SHIKHAR
ATIEE "
5. Inthe figure shown AB is parallel to DE. The absolute difference between angles x and y is

A35° B

DE
(A)0° (B)4° (©) 10° (D) 12°
6. In given figure find the values of x and y, if QS =RQ.

(A) x =36°, y =32° (B) x =45°,y=32°


(C)x=32°,y=45° (D) x=45°,y =26°
Subjective Type (4 Marks each, —1 for wrong answer)
7. According to the diagram, the value of (a + b) in degrees is

a
P

m;
M N

READ THE FOLLOWING THEOREMS :


(A) Congruence Theorems
Theorem 1 (SSS Congruence) : A P
If two triangles have corresponding equal sides then they are equal in all
aspects.
AABC = APQR B c Q R
Theorem 2 (SAS Congruence) : A P
If two triangles have two sides of one equal to corresponding two sides
of the other and the angles included by those sides are equal, then the
triangles are equal in all respects. B c Q R
AABC = APQR
Theorem 3 (AAS Congruence) :

If two triangles have two angles of one equal to corresponding two A A


angles of the other and any side of the first equal to the corresponding side A &
of the other, the triangles are equal in all respects. B T Q R
AABC = APQR
Theorem 4 (RHS Congruence) : A P

Two right angled triangles which have their hypotenuses equal, and one
side of one equal to one side of the other, are equal in all respects. Zfi Zfl
B C Q R
AABC = APQR
SHIKHAR : Indra Complex Vijaya Nagar, Chetakpuri Gwalior 0751-3551081
SHIKHAR
WTJEE
(B) Equiangular or Similar Triangles
Theorem 1 :
If two triangles are equiangular to one another, their corresponding sides
g
are proportional.
/A= 2P, /B = /Qand £C
= /R A
L AB_BC_CA
PQ
A
QR RP B c q R
Theorem 2 :
If two triangles have their sides proportional when taken in order, the triangles are equiangular to one
another, and those angles are equal which are opposite to corresponding sides.
P
AB_BC_CA
PQ QR RP A
= /A= /P, /B=/Qand Z/C= /R A
B Cc Q R
Theorem 3 :
If two triangles have one angle of the one equal to one angle of the other, and the sides about the equal
angles proportional, the triangles are similar. £
ZB=2Q A

e
A8 _BC AABC ~~ Al APQR A
PQ QR B c q R
Theorem 4 :
Consider two triangles ABC and PQR such that
P
BC_CA A i
£B = /Qand
QR RP
—=——, then
/\
A
/\ /?o\\
ZP= /A or ZP=180° — ZA B ¢ q R Q R
Theorem 5 :
In aright-angled triangle, if a perpendicular is drawn from the ,/
right angle to the hypotenuse, the triangles on each side of it
are similar to the whole triangle and to one another.
AABC ~ AAPB ~ ABPC
Theorem 6 :
The internal & external bisector of an angle of a triangle divides
the opposite side in the ratio of their sides containg the angle.

_BD_BD'_AB
DC CD' AC
Proportion Applied to Area
Theorem 7 :
P
The areas of similar triangles are proportional to the squares A
on corresponding sides.

[ABC] AB* BC* CA’ h!


= . B c Q R
[PQR]
PQ" QR* RP’ kI
‘Where [XYZ] denotes Area of Trianele XYZ.
SHIKHAR : Indra Complex Vijaya Nagar, Chetakpuri Gwalior 0751-3551081
SHIKHAR
ATIEE "
" PROBLEM SET-3 (3 Marks each, —1 for wrong answer)

1. InAABC and ADEF, ZA =50°, ZB =70°, £C =60°, 2D = 60°, ZE = 70°, £F = 50°, then AABC is
similar to
(A) ADEF (B) AEDF (C) ADFE (D) AFED

2. Triangle ABC is such that AB =9 cm, BC = 6 cm, AC = 7.5 cm. ADEF is similar to AABC,
If EF = 12 cm then DE is

(A)6cm (B) 16 cm (C) 18 cm (D) 15 cm


3. In a right angled AABC, £C = 90° and CD is the perpendicular on the hypotenuse AB, AB = c,
BC =a, AC=b and CD = p, then

w23 P
B,

® >
1 1 1

(C)pr=b2+c? C A

(D) PO
11,1 )

4. The areas of two similar triangles are in the ratio of 25 : 36. What is the ratio of their respective heights?
(A)5:6 (B)6:5 ©1:11 D)2:3
5. Inatriangle ABC as shown in the diagram, where AB = 5, AC = 10 and BD = 2 then perimeter of
triangle ABC is
A

B o c
(A)9 (B)21 (©)20 (D) 30
6. Let ABC be a triangle with ZB = 90°. Let AD be the bisector of ZA with D on BC. Suppose AC =6
cm and the area of the triangle ADC is 10cm?. Then the length of BD (in cm) is equal to

A >
(A) 5
B) =
(B) 10 ©) 3
o2 o2 3
Multiple Correct Answer Type (4 Marks each, —1 for wrong answer)
7. D, E, F are the mid points of the sides BC, CA and AB respectively of AABC. Then

Area of AABC _ 4 Area of ADEF -1


) “Area of AEDF () “Area of ABFD
Area of AABC
=2 _
(©) ‘Area of ABFD + Area of ADEC (D) Area of AAFE = Area of ADEC
READ THE FOLLOWING THEOREMS :
Two Important Results on Area
Theorem 1 :
Triangles on the same base and between the same parallels (hence, of the
same altitude) are equal in area.
Area of AABC = Area of AAPB

SHIKHAR : Indra Complex Vijaya Nagar, Chetakpuri Gwalior 0751-3551081


SHIKHAR
SUNTAR
IT-JEE
Theorem 2 :
If two triangles are equal in area and stand on the same base and on the
same side of it, they are between the same parallels.
If Area of AABC = Area of AAPB = AB | CP A B
SOME IMPORTANT POINTS AND THEIR MEANING :
(i) Altitude : The perpendicular dropped on any side of a triangle from the opposite vertex.
Orthocentre : The three altitudes of a triangle are concurrent and the point of concurrence is the
orthocentre.
(ii) Angle Bisectors : The bisectors of the interior angles of a triangle.
Incentre : The point of concurrence of the bisectors of the three interior angles. It is the centre
of the incircle.
Incircle : The circle that touches all the sides of a triangle internally.
(iii) Median : The line segment joining the mid-point of one side of a triangle with the opposite vertex.
éentmid : Point of intersection of all the three medians.
(iv) Perpendicular Bisectors of the sides of a triangle are concurrent.
Circumcentre : Point of concurrence of the Perpendicular Bisectors. It is the centre of the circle
passing through the three vertices of a triangle and that circle is known as cirumcircle.

Baudhayana Theorem (Pythagorean/Pythagoras' Theorem) : C


In a right-angled triangle the square described on the hypotenuse
is equal to the sum of the squares described on the other two sides. d
A B
In Right Angle Triangle ABC. ~ AC? = AB? + BC?

PROBLEM SET- 4 (3 Marks each, —1 for wrong answer)


1. IfIis the incentre of AABC, then ZBIC is equal to
1 1 1
(A) EA (B) 90° + EA (C) 90° — EA (D) A
2. Intriangle ABC, if P is orthocentre, then ZADC + ZAEB is

(A)90° (B) 180° (C)270° (D) 100°


3. Intriangle ABC, if AD is median, then which of the following is always true
(A) £ZBAD = Z/DAC (B) ZADB = ZADC
(C)£B=£C (D) BD=DC
4. If the sides of aright triangle are 9, 12 and 15 cm long, then the sum of squares of medians is
(A)227.5 (B)337.5 (C)537.5 (D) None of these
Multiple Correct Answer Type (4 Marks each, —1 for wrong answer)
5. Inatriangle ABC, ZA =90° and area of triangle is 3, where AB and AC are having integral lengths,
then BC can be
) ie (B) 35 © 13 (D) 37

SHIKHAR : Indra Complex Vijaya Nagar, Chetakpuri Gwalior 0751-3551081


SHIKHAR
SUNTAR
IT-JEE
Subjective Type (4 Marks each, —1 for wrong answer)
6. Let XOY be a triangle with ZXOY = 90°. Let M and N be the midpoints of legs OX and OY,
respectively. Suppose that XN = 19 and YM = 22. What is XY ?

X|
M

7. According to the diagram :

Let AC=3,BC =4, AB = 5 and line AB is parallel to CP then the length of perpendicular from P on
AB is
READ THE FOLLOWING THEOREMS :
QUADRILATERAL is a polygon with four sides.
Quadrilaterals may be classified as :
(a) Trapezium : A quadrilateral with a pair of opposite sides parallel is called trapezium.
If the lateral sides are equal then it is called Isosceles trapezium.
(b) Parallelogram : A quadrilateral with both the pairs of opposite sides parallel.
Properties :
. The opposite angles are equal
. The opposite sides are equal
. The diagonals bisect each other
. Each diagonal divides the parallelogram into two congruent triangles.
Some special types of parallelograms are :
(©) Rectangle : A parallelogram with any one angle measuring 90°.
Properties :
. All properties of a parallelogram and
. The diagonals are equal
. All angles are 90°
(d) Rhombus : A parallelogram with one pair of adjacent sides equal.
Properties :
. All properties of a parallelogram and
. All the sides are equal
. The diagonals are perpendicular to each other
. The diagonals divide the rhombus into four congruent triangles
. Each diagonal bisects the respective vertex angles
“(e) Square : It combines all the properties of a rectangle and rhombus.
Note : The special properties mentioned above are to be understood along with all the general
properties of parallelograms.

SHIKHAR : Indra Complex Vijaya Nagar, Chetakpuri Gwalior 0751-3551081


SHIKHAR
ATIEE "
Theorems of Parallelograms :
Theorem 1 :
The straight lines which join the extremities of two equal and parallel
straight lines towards the same parts are themselves equal and parallel.
AB = CD and AB | CD = BC = AD and BC | AD A B
Theorem 2 :
D
The opposite sides and angles of a parallelogram are equal to one another v
and diagonal bisects each other.
In parallelogram ABCD
AB = CD, BC = AD = AO = OC and BO = OD
Theorem 3 :
Parallelograms on the same base and between the same parallels are equal
in area.
Area of parallelogram PQRS = Area of parallelogram PQMN
PROBLEM SET- 5 : (3 Marks each, —1 for wrong answer)
1. Inatrapezium ABCD with bases AB and CD, where AB =52, BC =12, CD =39 and DA = 5. The
area of the trapezium ABCD, is
(A) 182 (B) 195 (©) 210 (D) 260
2. The figure formed by joining the mid points of the adjacent sides of a rhombus is a
(A) Square (B) Rectangle (C) Trapezium (D) None of these

3. Inthe given figure, ABCD is a parallelogram in which ZDAB = 75° and ZDBC = 60° then, ZBDC is
equal to
D) C

Al B
(A)75° (B) 60° (C)45° (D) 55°
4. Infig, ABCDis aparallelogram, AL L CD and AM L BC. If AB=12cm, AD =8 cmand AL = 6 cm, then
AM =
(A) 15cm DL ¢

(B) 9cm
8\ [6cm M
(C)10cm 2

(D) None of these A 12m B


5. D, E, F are mid points of the sides BC, CA & AB respectively of AABC, then area of parallelogram
BDEF is equal to

(A) % area (AABC) (B) %arca (AABC)

©) % area (AABC) (D) %area (AABC)


6. ABCD is a trapezium in which AB || CD. If ZADC = 2ZABC, AD =a cm and CD = b cm, then the
length (in cm) of AB is

(A)%+2b (B)a+b

©)
C E38 + b (D)a+
D 3
Zb

SHIKHAR : Indra Complex Vijaya Nagar, Chetakpuri Gwalior 0751-3551081


SHIKHAR
ST
IT-JEE
Subjective Type (4 Marks each, -1 for wrong answer)
7. Let ABCD be a convex quadrilateral with perpendicular diagonals. If AB =20, BC =70 and CD =90,
then find the value of DA.
8. Inrectangle ABCD, point X and Y are the mid point of AD and DC, respectively. Lines BX and CD
when extended intersect at E, lines BY and AD when extended intersect at F. If the area of ABCD is

60 and area of ABEF is p then ‘1% is

The Circle, Definitions and First Principles Chords


Theorem 1 :
If a straight line drawn from the centre of a circle bisects a chord, then it
cuts the chord atight angle. Conversely, if it cuts the chord at right angle,
it bisects it.
Theorem 2 :
There exists a unique circle which passes through three non collinear points

Theorem 3 :
If from a point within a circle more than two equal length of straight lines
can be drawn to the circumference, that point is the centre of the circle.

OA = OB = OC = O is the centre of circle.

Theorem 4 :
Equal chords of a circle are equidistant from the centre.
AB
= CD, OP
L AB and
OQ L CD = OP = OQ
Conversely,

Chords which are equidistant from the centre are equal.

OP = 0Q, OP L AB and OQ L CD = AB =CD

Theorem 5 :

If we are having two chords of unequal length in a circle, then the chord
which is having greater length is nearer to the centre of circle . Conversely,
out of two chords which is nearer to the centre of the circle is having greater
length.

OP > 0Q = CD < AB

Theorem 6 : R
east
If from any internal point, not the centre, straight lines are drawn to the e b o
circumference of a circle, then the greatest is that which passes through
N2

the centre and the least is the perpendicular to that diameter. Greatest &

SHIKHAR : Indra Complex Vijaya Nagar, Chetakpuri Gwalior 0751-3551081


SHIKHAR
WTJEE
PQ is the greatest and RS is the least chord passing through A. And R Q

of any other two such lines the greater is that which subtends the greater \ E
angle at the centre. ‘
P
ZSOR > ZPOQ = SR > PQ

Theorem 7 :

If from any external point straight lines are drawn to the circumference of a circle, the greatest is that
which passes through the centre and the least is that which when produced passes through the centre.
And of any other two such lines, the greater is that which subtends the greater angle at the centre.

. /\

Greatestw

ZCOD > ZAOB


PD>PB

Angles in a Circle
Theorem 1 :
The angle at the centre of a circle is double of an angle at the circumference
standing on the same arc.

ZAOB = 2ZAPB
Theorem 2 :
Angles in the same segment of a circle are equal.

ZAPB = ZAQB
Converse of Theorem 2 :
Equal angles standing on the same base and on the same side of it, have
their vertices on an arc of a circle, of which the given base is the chord.

If ZAPB = ZAQB = AB is a chord of circle and

A, P, Q, B are concyclic.

Theorem 3 :
The opposite angles of any quadrilateral inscribed in a circle are together
equal to two right angles.

ZP + ZR =180° and £Q + £S = 180°


Converse of Theorem 3 :
If a pair of opposite angles of a quadrilateral are supplementary, then its
vertices are concyclic.
If ZP + ZR = 180° and £Q + £S = 180°
= Then point P, Q, R and S are concyclic.

SHIKHAR : Indra Complex Vijaya Nagar, Chetakpuri Gwalior 0751-3551081


SHIKHAR
ST
IT-JEE
Theorem 4 : p

The angle is a semi-circle is a right angle. /A\

ZAPB = 90° Av B

Theorem 5 :
In acircle, arcs which subtend equal angles, either at the centres or at the
circumferences, are equal. R S
ZAOB = #POQ forfig. (i) (‘ “
= AB=PQ A o
ZARB = £PSQ for fig. (ii) BP
ey Fig. (i) Fig. (ii)
= AB=PQ
Theorem 6 :
In acircle, angles either at the centre or at the circumference, which stand
on equal arcs are equal. RS

AB=PQ for fig. (i)


= ZAOB = ZPOQ A | )

AB=PQ for fig. (ii) Fig. () Fig, (i)

= ZARB = £PSQ
Theorem 7 :
M P
In equal circles, arcs which are cut off by equal chords are
equal, the major arc equal to the major arc and the minor to D
the minor. A B 4
If Circle (i) = Circle (i) N C
AB = CD Circle (1) Cirele (2)

— AMB = CPD and ANB = CQD


Theorem 8 : M )}
In equal circles chords which cut off equal arcs are equal.
Circle (i) = Circle (ii) D
A B
AMB = CPD and ANB = CQD N o
= AB = CD Circle (1) Circle (2)

Theorem 9 :
If a line segment joining two points subtends equal angles at two other points lying on the same side
of the line containing the line segment, then these four points lie on a circle (i.e. they are concyclic).
PROBLEM SET- 6

1. Ois the centre of the circle. If chord AB = chord CD, then x =


(A) 70°

(B) 50°

(C) 55°
(D) 45°

SHIKHAR : Indra Complex Vijaya Nagar, Chetakpuri Gwalior 0751-3551081


SHIKHAR
NT-JEE
2. Ois the centre of the circle. AB and CD are two chords of the circle. OM L AB and ON L CD.
If OM = ON = 3 cm and AM = BM = 4-5 cm, then
CD =
(A) 8 cm 4
(B) 9 cm \
el B
(C) 10 cm @

(D) None of these

3. Inthe given figure, find the value of x. (O is the centre of the circle)

(A) 68°
(B) 63°
(C) 252°
(D) None of these

4. Find the value of x in the given figure.


(A) 45°
(B) 28°
(C) 60°
(D) 55°
5. Inthe given figure, ABCD is a cyclic quadrilateral in which ZBAD =120°. Find ZBCD.
(A) 240°
(B) 60° A
() 120° \ /
(D) 180° ¥
6. In the given figure, two chords AB and CD of a circle intersect each other at a point E such that
ZBAC = 45°, ZBED = 120°. Then find ZABD.
(A) 15°
(B) 30°
(C) 45°
(D) 60°
7. Inthe given figure, AABC is inscribed in a circle with centre O. If ZACB = 65°, find ZABC.
(A) 25° A

B Ne
(C) Cannot be determined v

(D) None of these


8. Infigure, Ois centre, then ZBXD =

(A)65° (B) 60° (©)70° (D) 55°


SHIKHAR : Indra Complex Vijaya Nagar, Chetakpuri Gw: r 0751-3551081
SHIKHAR
NT-JEE
9. Inthe figure shown, chord ED is parallel to diameter AC of a circle. If ZCBE = 60°, then ZDEC must be

(A)15° (B) 30° (©€)10° (D)20°


10. In figure, PQRS is a cyclic quadrilateral. Its diagonals PR and QS intersect each other at T. If
ZPRS = 80° and ZRQS = 50°, calculate ZPSR.

(A)30° (B) 50° (C)70° (D) 90°


11. Infigure, PQis a diameter of acircle with centre at O and OR L PQ, where R is a point on the circle. If
S is another point on the circle such that ZRPS = 32°, then ZQRS is

(A)13° (B)26° (D) None of these


12. Infigure, ZBDC is equal to

(A)95° (B) 105° (C) 100° (D) 110°


13. Find ZASR, in the given figure

(A)52° (B)78° (C) 102° (D) 100°


SHIKHAR : Indra Complex Vijaya Nagar, Chetakpuri Gw: r 0751-3551081
SHIKHAR
ST
IT-JEE
14. In figure, CDEF is a cyclic quadrilateral, DE and CF are produced to A and B respectively such that AB
| CD. If ZFED = 80°, find ZFBA.

(A)30° (B) 60° (C) 80° (D) 100°


15. In the given figure, O is the centre of a circle and BD is a diameter. AB and AC are tangents touching
the circle at B & C respectively. If ZBAC = 70° then ZOBC is
B

(A) 30° (B) 35° (C)40° (D) 45°


16. In the given figure, O, O' are centres of two circles intersecting at B and C. ACD is a straight line. Find x.

(A)130° (B) 50° (C)40° (D) 65°


17. ABCD is a cyclic quadrilateral inscribed in a circle with the centre O. Then ZOAD is equal to

(A) 30° (B) 40° (C)50° (D) 60°

Tangency

Theorem 1 : .
The tangent at any point of a circle is perpendicular to the radius drawn
to the point of contact.
Theorem 2 :
Two tangents can be drawn to a circle from an external point.

PM
= PN
Theorem 3 :
If two circles touch one another, then the centres and the point of contact lie on a straight line.

SHIKHAR : Indra Complex Vijaya Nagar, Chetakpuri Gwalior 0751-3551081


SHIKHAR
ATIEE "

@
00'=r,+r, 00'=|r, -1

Theorem 4 (Alternate Segments Theorem) : P, M


The angles made by a tangent to a circle with a chord drawn from the point
of contact are respectively equal to the angles in the alternate segments
of the circle.
ZMTB = ZTPM
POLYGON
() Rectilinear Figure or Polygon : A plane figure bound by (three or more) lines.
(Rectilinear figure may be closed or open while polygon is closed)
(i) 1f a polygon has 'n' sides, then

(n-1)
(a) The number of diagonals is d = = n2 =i

(b) The sum of its interior angles = (2n — 4) right angles


or =(n-2) x 180°
(i) Concave Polygon : A polygon with at least one interior angle reflex.
Convex Polygon : If all interior angles are less than 180°, then the polygon is convex.
(iv) Sum of all the exterior angles of a polygon whose sides are produced in order is 360°.
(v) Regular Polygon is one in which all sides and angles are equal. In a regular polygon of 'n’ sides, the
value of each exterior angle is given by :

360°
Ex. Angle =
n
and hence the value of each interior angle is :

360°
Int. Angle = 180° —

as the sum of the interior and the exterior angles at any vertex is 180°.
PROBLEM SET- 7 (3 Marks each, —1 for wrong answer)

1. Ina regular polygon, if interior angle is 144°, then number of sides is

(A)6 (B)8 © 10 D) 12
2. If four sides of a quadrilateral ABCD are tangential to a circle, then
(A) AC + AD = BD + CD (B) AB + CD = BC + AD

(C) AB + CD = AC + BC (D) AC + AD = BC + DB
3. If TP and TQ are two tangents to a circle with centre O so that ZPOQ = 110°. then ZPTQ is equal

to

(A) 60° (B) 70° (C) 80° (D) 90°


4. The length of the tangent drawn from a point 8 cm away from the centre of a circle of radius 6 cm is

(A) J7cm (B) 247 cm (C) 10 cm (D) 5 cm


SHIKHAR : Indra Complex Vijaya Nagar, Chetakpuri Gwalior 0751-3551081
SHIKHAR
SUNTAR
IT-JEE
5. An equilateral triangle XYZ is inscribed in a circle with centre O. The measure of XOY is
(A) 60° (B) 120° (C) 45° (D) 75°
6. In figure, PA and PB are the two tangents drawn to the circle. O is the centre of the circle. A and B are the
points of contact of the tangents PA and PB with the circle. If ZOPA = 35°, then ZPOB =
(A)55° N
(B) 65° o
(C)75°
(D) 85° p
A
7. If the sum of all angles except one of a convex polygon is 2180°, then the number of sides of the

polygon is
(A) 19 (B)17 ©)15 (D) 13
Subjective Type (4 Marks each, —1 for wrong answer)
8. Two circles touch externally. The sum of their arcas is 130w sq. cm. and the distance between their

centres is 14 cm. Find the product of radii of the circles.


READ THE FOLLOWING THEOREMS :
MENSURATION |
For a goemetrical figure, plane or solid :
Perimeter is the length of its boundary and its unit is the same as that of length. Area is the measure
of the surface enclosed by its boundary. Its unit is square units.
Volume of a solid is the measure of the part of space occupied by it. Its unit is cubic units.
) Triangles :
Considering the base as 'b'
Corresponding altitude as 'h'

1
Area = —xbxh
5

2x A 2 x Area
Longest Altitude = _ZXATR —
g ortest Altitude = e
Shortest Base Longest Base

Heron's Formula :
If a, b, ¢ are the sides of a triangle

Area = 3(S-a)(S-b)(S-0) where §= %%b


If 'a' is a side of an equilateral triangle,

Area =
2
Altitude = -
Va

If 'a" is the measure of the equal sides and 'b' the third side of an isosceles triangle,

i} \4a u
Area = Xb 4% —b? Altitude =
2
Perimeter in all cases is the sum of the three sides
(ii) Quadrilaterals :
Rectangle :
If'¢', 'b', 'd' are the length, breadth and diagonal, then

SHIKHAR : Indra Complex Vijaya Nagar, Chetakpuri Gwalior 0751-3551081


SHIKHAR
SUNTAR
IT-JEE

Area={ x b Perimeter = 2({ + b) d=v2+b2

¢ Square:
If each side is 'a'

Area = a2 Perimeter = 4a Diagonal (d) = fia Area= 7

. General Quadrilateral :
In any genetal quadrilateral ABCD if diagonal AC = d
perpendiculars from B and D to AC are h, and h,, then

Area =%d(hl +h,)

¢ 'Rhombus :
If the diagonals are d,, d, and each side 'a', then

2 2
Areaz%cll xd, at= (fi) +(9_2_]

. Note :
For any quadrilateral whose diagonals d, and d, are perpendicular to each other,

Area = %d, xd,

. Parallelogram :
If'b" any side and 'h' the corresponding altitude
Area=b x h
. Trapezium :
If the parallel sides are a and b and the distance between them h,

Area :%(a+ b)h

10 CIRCLE, SECTOR AND SEGMENT :


(i) Letin a circle of centre O
radius = r AB a chord
ZAOB = 6 (the angle subtended by arc ACB at the centre)
OACBO the minor sector,
ACB the minor segment,
BDAB the major segment.
. Circumference of the circle = 2nr
e Area of the circle = nr?
. Perimeter of semi-circle = nr + 2r

. Length of arc ACB(/fiE) ={=10


where 0 is in radian

1
. Area of sector OACBO = Erze

1 1 1
[Area of sector OACBO may also be : Erze = ?‘9 r= E[Ar]

SHIKHAR : Indra Complex Vijaya Nagar, Chetakpuri Gwalior 0751-3551081


IT-JEE
Perimeter of sector OACBO = OA = OB + Arc ACB =
r+r+r0=2r+ro
Area of the minor segment ACBA = Area of sector OACBO — Area of AOAB

;[lrz(')—lr2 sin6:|
2 2

(i) Ring:
Part of the plane between two concentric circles of different radii
(r, and r, where r; > 1,),
Area of Ring = nr? — nr} = n(r; + 1,)(r; — 1)
(iii) If two circles of radii r; and r, withr; > r, touch each other
o Externally, the distance between their centres = (1, + r,)
o Internally, the distance betweeen their centres = (r, —r,)
(iv) In the case of rotating wheels
o Distance moved by a wheel is one rotation is the circumference of the wheel,
o Nimibet of rotationi= Distance Travelled
Circumference of the wheel
() Rotation of the hands of a clock : )
o Angle described by the minute hand of a clock in 1 hour (60 minute) i.e. one rotation = 360°.
o Angle described by the hour hand of a clock in 12 hours (i.e. one rotation) = 360°.
PROBLEM SET- 8 (3 Marks each, —1 for wrong answer)
1. The area of a rhombus is 2016 sq cm and its side is 65 cm. The lengths of the diagonals (in cm) are
(A) 125,35 (B) 126,32 (C) 132,26 (D) 135,25

2. A square and an equilateral triangle have the same perimeter. If the diagonal of the square is 1242 cm,
then the area of the triangle is
(A) 2443 em? (B) 247 em’ (€) 6443 em*(D) 3243 em’
3. Inthe figure, when the outer circles all have radii 'r', then the radius of the inner circle will be

1 2
A)V2r (B) (V2 -1 (C)E D) WZ+1r
4. Awireis in the form ofa circle of radius 35 cm. If it is bent into the shape of a rhombus, what is the side
of the rhombus? s
(A)32cm (B) 70 cm (C)55em (D) 17 cm
5. There are two concentric circles whose areas are in the ratio of 9 : 16 and the difference between their
diameters is 4 cm. What is the area of the outer circle?
(A) 321 cm? (B) 64n cm? (C) 36m em? (D) 487 cm?
SHIKHAR : Indra Complex Vijaya Nagar, Chetakpuri Gwalior 0751-3551081
SHIKHAR
ATIEE "
6. ABCD isa square, four equal circles are just touching each other whose centres are the vertices A, B, C,
D of'the square. What is the ratio of the shaded to the unshaded area within square?

8 3 5 6
A) 7 ®) 17 © 1 D7
7. A wire in the shape of an equilateral triangle encloses an area of S sq. cm. If the same wire is bent to
form of a circle. The area of the circle will be :

7s? 2 38 38 3438
(A) ) B © ® =
8. Inthe given figure, the diameter of the biggest semi-circle is 56 cm and the radius of the smallest circle
22
is 7 cms. The area of the shaded portion is : (Take = 7)

A B
C
(A) 482 cm® (B) 462 cm’ (C) 654 cm? (D) 804 cm®
Subjective Type (4 Marks each, -1 for wrong answer)
9. The diameter of a cycle wheel is 28 cm. How many revolution will it make in moving 13.2 km ?
10. The minute hand ofa clock is 10 cm long. Find the area (in cm?) of the face of the clock described by the
minute hand between 9 A.M. and 9.35 A.M.

PROBLEM SET-9 (3 Marks each,-1 for wrong answer)


a b
1. If (2> +b%?=(a* + b*)* and ab # 0 then the numerical value of g'*; is equal to -

3 2 4
w5 ® 3 ©1 Dy
2. Which of the following is the smallest positive integer which gives the remainder 3 when divided
with 4, 6, 8, and 10?
(A)73 (B) 103 ©)123 (D) 243
3. Theratio of total area of the rectangle to the total shaded area

4-n
@ = ® B) o © T o= 2
4. Theexpression 3(a’+ 1)*+2(a— 1)(a® + 1) = 5(a— 1)* - 4(0.75a" + 3a — 1) when simplified reduces to
(A)22° —a° (B)2a’ -2’ (€) 24° (D) 2a°
5. Ifx+y=aand X2+ y2= b, then the value of (x3 + ys), is

(A)ab B) 2 +b ©a+b® D) = —
SHIKHAR : Indra Complex Vijaya Nagar, Chetakpuri Gwalior 0751-3551081
SHIKHAR
WTJEE
6. Which one of the following does not reduce to sinx for every x, wherever defined, is

tanx sinx sin® xsecx .


D) All reduce to sinx
@) secx ® sec’ x —tan’ x © tanx (D) All redu

7. Solution set of the equation 32 — 2.354x46 L 3200 _ ) s


(A){-3,2} (B) {6,-1} (©) {-2,3} (D) {1,-6}
8. Whatis the area of an equilateral triangle inscribed in a circle of radius 4 cm ?
2 ;)
(A)12cm? (B) 943 cm” (©) 83 em’ (D) 123 cm
? 2x+7 4-6
9. The equation I Zxil, X +1=0 has the roots -
x-1 3 x-1
(A)4and 1 (B) only 1 (C) only 4 (D) Neither4 nor 1

10. The expression + 12+6\/§ + 12—6J§ simplifies to

(a4 ®) 243 © 33 D)6


PROBLEM SET-10
1. State whether the following statements are True or False : 2x1

) ) 1 1 1 1 1 1 Y
@) Ifx,y,zareall different
real numbers, then Gy B G-2f + (ZT me = s s ez # =

(ii) There exist natural numbers, m & n such that m* = n? + 2010.


Straight Objective Type (3 Marks each, —1 for wrong answer)
2. The angles of a triangle are in the ratio of 2 : 3 : 4, the measurement of greatest angle is -
(A)30° (B) 60° (C) 100° (D) 80°
3. Let ABCD be a trapezium, in which AB is parallel to CD, AB = 11, BC =4, CD = 6 and DA = 3.
The distance between AB and CD is
(A)2 (B)2.4
(C)2.8 (D) not determinable with the data
4. Ifin the figure AB=4,BC =6,CA =8then AZ+BX +CY is:-
(A) 18 A

©6
©e6 e fi
(D) 12
5. If AB and CD are perpendicular diameters of circle Q. and ZQPC =60°. then the length of

PQ divided by, the length of AQis :-

(A) e
V3 (B) 5
V3

®5 2
Multiple Correct Answer Type (4 Marks each, —1 for wrong answer)

1 1 1 1 1
6. Let T= - + — + then -
3-V8 BT V1-V6 Vo5 V542
(AT<1 B)T=1 ©)1<T<2 D) T<2

SHIKHAR : Indra Complex Vijaya Nagar, Chetakpuri Gw: r 0751-3551081


SHIKHAR
NT-JEE
7. If ‘a’ and ‘b’ are two distinct prime numbers lying between 1 and 10, which of the following can be the
sum of ‘a’ and ‘b’-
(A)5 B)6 ©7 D)8
8. The points A, B, C, D, E are marked on the circumference of a circle in clockwise direction such that
ZABC = 130° and ZCDE = 110°. The measure of ZACE in degrees is greater than or equals to ;
(A) 50° (B) 60° () 70° (D) 80°
9. In the adjoining figure ACB is a quadrant with radius 'a’. A semicircle is drawn outside the quadrant
taking AB as a diameter. Find the area of shaded region can not be :

o2
(A) %(anaz) (B) %(na2 -a?) (© % (D) 2
Matrix Match Type (8 Marks each, -1 for wrong answer)
10. Column-I Column-IT
(A) Number of solution of the equation vx +6 =x is ® 1
(B) The number of integral pair(s) (x, y) whose sum is equal to Q 2
their product is
1 1
© x= s N
and ¥ =—===then the value of (x - y)*eq
y)? equals R)R) 3

1
(D) Let x =1+————————— then the value of (2x — 1)? equals S 4
24—
1+

ANSWER KEY
PROBLEM SET- 1
1. 2. 3 a 5. 6.
B c A B c 93"
PROBLEM SET-2
1. 2. 3. a 5. 6. 7.
B B c B B D 70
PROBLEM SET-3
1. 2. 3. a 5. 6. 7.
D c D A B D ABCD
PROBLEM SET-4
1. 2. 3. a. 5. 6. 7.
B B D B cD 26 24
PROBLEM SET-5
1. 2. 3. a. 5. 6. 7. 8.
c B c B A B 60 6
PROBLEM SET-6
1. 2. 3. a 5. 6. 7. 8 9 10
c B B B B A A c B B
1. 12. 13 14 15. 16. 17.
A B c c B A D
PROBLEM SET-7
1. 2. 3. a 5. 6. 7. 8.
c B B B B A c 33
PROBLEM SET-8
1. 2. 3. a 5. 6. 7 8. 9 10
B c B c B B D B 15000 | 183.33
PROBLEM SET-9
1. 2. 3. a. 5. 6. 7. 8 9. 10
B c B A D D c D c D
PROBLEM SET- 10
1. 2. 3. a 5. 6. 7. 8. 9 10
0T, () F D B B B BD ACD AB ABD P.QQR
NOTE: REFERENCE BOOKS - RD Sharma (Class 10th), NCERT Textbook(Class 10th), NCERT Exemplar (Class 10)
Further reading:- MTG Mathematics(Class 10th), Pearson Mathematics(Class 10th), NTSE and NSEJS PYQs

SHIKHAR : Indra Complex ya Nagar, Chetakpuri Gwalior 0751-35510


SHIKHAR
NT-JEE

ALGEBRA PRACTICE PROBLEM SETS


PROBLEM SET-1
1. If the polynomial
2x® + ax® + 3x — 5 and x* + x” — 4x + a leave the same remainder when divided by x —
2, find the value ofa.

2. Find
the value of p and q so that x* + px® + 2x” — 3x + q is divisible
by x* — 1.

3. If the polynomial P(x) = 2x* + x> — 5x” — x + 1 is divided by the polynomial Q(x) = x* — x then the
remainder is a linear polynomial R(x) = ax + b. Then (a + b) equals :
A)-2 ®)-1 ©1 D)2
4. ‘What must be subtracted from x* — 6x” — 15x + 80 so that the result is exactly divisible by x* +x — 12.

5. Apply the division algorithm to find the quotient and remainder on dividing p(x) = x* — 3%’ + 4x + 5 by
g =x+1-x

6. Obtain all the zeros of 3x* + 6x° — 2x” — 10x — 5, if two of its zeros are Eand—‘jg.

7. ‘What must be added to x* — 3% — 12x + 19 so that the result is exactly divisible by X’ +x —6 ?

8. What must be subtracted from x* + 2x* — 13x” — 12x + 21 so that the result is exactly divisible by
® - Ax+3?

9. If 4x* - 3x® - 3x>+x - 7 is divided


by 1 - 2x then remainder
will be :
57 59 55 55
)A) =s (B) -=Py ©C) =Py D) -—Py
(D)

10. A student used division to divide a polynomial p(x) by x — 3 and got a remainder of 5. Which of the
following statements about p(x) must be true?
(A) x - 3 is a factor of p(x). (B) x5 is a factor of p(x)
(C) The value of p(-3) is 5. (D) The value of p(3) is 5.
PROBLEM SET-2
1. Find the value of a if the division of ax® + 9x> + 4x — 10 by (x + 3) leaves a remainder 5.
@)1 B)2 ©3 )4
2. If (x + 1) and (x — 2) are factors of x* + ax” — bx — 6, then find the values of a and b respectively.
(A)2,3 (B)3.5 ©)s,3 D)2,5

3. If (5%* + 14x + 2)* — (4x* — Sx + 7)? is divided by (x* + x + 1) then, quotient 'q' and remainder 'r' are
given by :
(A)q=(¢+19x - 5),1=0 (B)q=9(*+19x - 5),r=0
(C)q=(+19x-5),r=1 (D)q=9(x*+19x—-5),
1= 1

4. If ("' + 1) is divided by (x + 1), then the remainder is :


@ao (B)2 ©1 (D) 10

5. When (x’ - 2x* + px — q) is divided


by (x* — 2x — 3), the remainderis (x — 6). The values
of p and q are :
(A)p=-2,9=-6 B)p=2,9=-6 ©p=-2,9=6 D)p=2,9=6
6. Find the value ofk, if (x + 2) exactly divides x> + 6x> + 4x + k.
A4 (B)6 ©-8 (D)-10

7. Which one of the following is a factor of x* — 5x* + 53 — 10x +24?


(A) (x+2) B) (x+1) ©x 2 (D) none of these
SHIKHAR : Indra Complex Vijaya Nagar, Chetakpuri Gw: r 0751-3551081
SHIKHAR
NT-JEE

8. If (x + k) is a common factor
of (x* + px + q) and
(x> + Ix +m), then the value
of k is

@1+p ®m-q © 2
m-q
D1-p
9. (& —x®+x" — 1) is divisible by
(A) (x+ 1) butnot by (x — 1) (B) (x— 1) but not by (x + 1)
(C) both (x + 1) and (x— 1) (D) neither (x—1) and (x + 1)

10 x4+ 2x% +kis divisible by (x + 1), then the value ofk s :


-3 ®)-2 ©1 )2
PROBLEM SET-3

1. Ifm:—4m+l:0,flmfllevxlueuf(m3+%)is:
m’
(A)48 (B)s52 (©) 64 (D) None of these

2. The factors of (x* + 16) are :


AEHHEE-4) @)Y (Ox(x+2)(x'+4) (D)do not exist

3. The factors of x(y” - 2°) + y(z — x%) + z(x* — y?) are :


A) x=y)-2(z-%) B) (x+y)y +2)(z+x)
© y-x)E-y)(x-2) D) (x +y)z-y)(x~7)

4. The LCM of (a® + b%) and (a* — b*) is :


(A) (@ +b”) (2> + ) (a—b) (B) (2’ +b%) (a’+b") (a+b)
(C) (@ +b%) (a® + b*+ab) (a+ b) (D) (@ +b%) (2>~ b%) (a—b)

5. The GCD of (2% — 4x), (3x* — 12x%) and (2%’ — 2x" — 4x’) is :
(A)x(x-2) (B) 2x(x—2) (©)2x(2-x) (D) 2x(2 +x)

L 1 1 1 .
& Sl e Boha) coaEb)
Ao @31 ©3 (D) none of these

1 1 2 4 ) .
7. Express a-% + %) (l+x2) ' (l+x4) as a rational expression :

a1 ®) 11-x*L O 8 (D) none of . these

2 2 2
8. If a, b, c are all positive, then the minimum value of the expression @+atr DO +DHD(EH et is:
abe
(A)3 ®)9 ©)27 ™)1

9. l.fa+b+c=3‘a’+b2+c2=6mdl+%+l=lwhexea,b,careal.lnon—zero,thenabcis:
a c

1 2 3
( A) )3— ( B) )3= ( C) )2= ()]
D)1

10. 2% =4 = 8" and xyz = 288, then value of L+L+L is:


2x 4y 8z
11 11 29
( A) )12— (B) fulal
% ( C) )96
= (D) D none of f thes these

SHIKHAR : Indra Complex Vijaya Nagar, Chetakpuri Gwalior 0751-3551081


SHIKHAR
IT-JEE
PROBLEM SET-4
1 If the lines given by 3x + 2ky =2 and 2x + 5y + 1 = 0 are parallel, then the value of k is
=5 2 15 3
(S
A) — ®) =3 © C) — ® D) 3=

Solve the following pair of linear equations:


20x +47y =110
47x + 21y = 162

If 2x + y=23 and
4x — y = 19, find the values of 5y — 2x and Y o
X

Solve the following pair of linear equations:


Bx+67y=-24
67x + 43y =24

Solve the following pair of linear equations:


2xy 3
X+y 2
Xy -3
=—,x+ty#0,2x—y#0
2x-y 10 Y Y

Ifx + 1 is a factor
of 2x* + ax” + 2bx + 1, then find the values of a and b given that 2a —3b = 4.

It can take 12 hours to fill a swimming pool using two pipes. If the pipe of larger diameter is used for 4
hours and the pipe of smaller diameter for 9 hours, only half the pool can be filled. How long would it
take for each pipe to fill the pool separately?

A motor boat can travel 30 km upstream and 28 km downstream in 7 hours. It can travel 21 km
upstream and return in 5 hours. Find the speed of the boat in still water and the speed of the stream.

A shopkeeper sells a saree at 8% profit and a sweater at 10% discount, thereby, getting a sum Rs 1008.
If she had sold the saree at 10% profit and the sweater at 8% discount, she would have got Rs 1028.
Find the cost price of the saree and the list price (price before discount) of the sweater.

10. Vijay had some bananas, and he divided them into two lots A and B. He sold the first lot at the rate of
Rs 2 for 3 bananas and the second lot at the rate of Re 1 per banana, and got a total of Rs 400. If he had
sold the first lot at the rate of Re 1 per banana, and the second lot at the rateof Rs 4 for 5 bananas, his
total collection would have been Rs 460. Find the total number of bananas he had.
PROBLEM SET-5

1. The length of a hall is 5 m more than its breadth. If the areaofiheflonrofthehn]lix&lm’,whfltm


the length and the breadth of the hall?

2. Out of a group
of swans, % times the square root of the total number are playing on the shore of a tank.

The two remaining ones are playing, in deep water. What is the total number of swans?

The hypotenuse of a right triangle is 25 cm. The difference between the lengths of the other two sides
of the triangle is 5 cm. Find the lengths of these sides.

Swati can row her boat at a speed of 5 ki/h in still water. If it takes her 1 hour more to row the boat
5.25 km upstream than to return downstream, find the speed of the stream.

The sum
of the squares of two positive integers is 208. If the square of the larger number is 18 times the
smaller number, find the numbers.

SHIKHAR : Indra Complex Vijaya Nagar, Chetakpuri Gw: r 0751-3551081


IT-JEE
6. The length of a rectangle is greater than twice its breadth by 2 cm. The length of its diagonal is 13 cm.
Find the length and breadth of the rectangle.

A car covers a distance 300 km with same speed, it will cover the same distance in one hour less if its
speed is increased
by 10 km per hour. Find the speed of the car.

A cistern can be filled by two pipes in 33% ‘minutes; if the larger pipe takes 15 minutes less than the

smaller to fill the cistern, find in what time it will be filled by each pipe separately.

A train travels at a certain average speed for a distance of 54 km and then travels a distance of 63 km at
an average speed of 6 km/h more than the first speed. If it takes 3 hours to complete the total journey,
what is its first speed ?

10. If Zeba were younger by 5 years than what she really is, then the square of her age (in years) would
have been 11 more than five times her actual age. What is her age now?

PROBLEM SET- 6

1. If the sum
of first n even natural numbers is 420, find
the value ofn.

2. The denominator of a fraction is one more than twice the numerator. If the sum of the fraction and its
reciprocal is 2% find the fraction.

A two—digit number is four times the sum and three times the product of its digits. Find the number.

One-fourth of a herd of camels was seen in the forest. Twice the square root of the herd had gone to
mountains and the remaining 15 camels were seen on the bank of river. Find the total number of
camels.

A natural number when increased by 84 equals 736 times its reciprocal. Find the number.

Two trains leave a railway station at the same time. The first frain travels due west and the second train
due north. The first train travels 5 km/hr faster than the second train. If after
two hours, they
are 50 km
apart, find the average speed of each train.

An express train takes 1 hour less than a passenger train to travel 132 km between Mysore and
Bangalore (without taking into consideration the time they stop at intermediate stations). If the average
speedof the express train is 11 km/hr more than that of the passenger train, find the average speeds of
the two trains.

‘While boarding an aeroplane, a passenger got hurt. The pilot showing promptness and concern, made
amrangements fo hospitalise the injured and so the plane started late by 30 minutes to reach the
destination, 1500 km away in time, the pilot increased the speed by 100 km/hr. Find the original
speed/hour of the plane.

A motor boat whose speed in still water is 18 km/hr takes 1 hour more to go 24 km up stream than to
return down stream to the same spot. Find the speed of the stream.

10. Seven years ago Varun's age was five times the square of Swati's age. Three years hence Swati's age
will be two fifth of Varun's age. Find their present ages.

SHIKHAR : Indra Complex Vijaya Nagar, Chetakpuri Gwalior 0751-3551081


SHIKHAR
SUNTAR
IT-JEE
TRIGONOMETRY PROBLEM SET
PROBLEM SET- 1
1. If sinf= % find all the other ratios.
5

2 40
2. If tan6= IT' find the value of ik
5 c0s0 + cosech

3. If cosec A =§ , then prove that tan? A — sin? A = sin“A sec? A.

4. In A ABC, right angled at B, AC + AB = 9 cm and BC = 3 cm. Determine the value of cot C, cosec C.

5. Find value of sin 6. sec 6 .cosec 6. cos 6

Answers
5 209
1. = 2.
3 229
4. cotC:g.cosecC:§ 5. 1
4 4
PROBLEM SET- 2
1. Evaluate sin? 45°. cos? 45° + sin? 60° . cos? 60°.

2. Find the value of x from the equation, x tan 45°. sec 60° = cot? 30°.

3. If A= 60° and B = 30°, verify that sin A cos B + cos A sin B =sin (A + B)

4. If 8 is an acute angle and sin 8 = cos 6, find the value of 2 tan? 6 + sin2 6 — 1.

5. Evaluate
(@) sin? 60° + cos? 45° (b) 3 cos?30° + tan? 60°
(c) 4 sin?45° + 3tan? 30° + 8 sin 45°. cos 45° (d) 2sin2 30° — 3 cos? 60° + cot? 30°
© sin30°.cos60° ® sin60°.tan60°
tan45°.cot45° c0s30°.cot30°
1-tan”30°
(9) .
1+tan“30°
(h) 4(sin* 30° + cos' 60°) — 3(sin? 45° — sin 90°)

Answers
—7 2. X==3 4. =3
16 2 2

S, () 3 (b) 2 () 7 (d) u (e) L (U


. 4 4 4 4

(9) 3L (h)
h; 2

PROBLEM SET- 3
1 Simplify cot70’ . €052
tan20® sin38°

2. Prove that cos 72° . cos 18° - sin 72°. sin 18°=0

SHIKHAR : Indra Complex Vijaya Nagar, Chetakpuri Gw:


IT-JEE
3. If sin A = cos B, what is A + B equal to?

4. Evaluate : cos(40° + 6) —sin (50° - 6)

5. Evaluate : tan 13° .tan 21°. tan 69°. tan 77°

Answers

1. 2 3. 90° 4. 0 5. 1
PROBLEM SET- 4
1. Express cos 6 in terms of sin 6.

2. Express tan 6 in terms of cos 6.

3. Prove that, (sin 6 + cos 62 =1+ 2 sin 6 cos 6

4. Prove that cot? 6 + cosec? 6 = cosec* 6 — cot* 6

5. Prove that tan® + fan® =2cosech


secO-1 secH+1

6. If x=acos 6, y =asin 6, what is the value of x? + y??

7. If x = cosec 6 — cot 8, y = cosec 6 + cot 8, find xy.

Answers

1. cos@=+/1-sin’0 2. tang=YL
O ¢ a 7. xy=1
cosB

PROBLEM SET- 5
1. Find the angle of elevation of the sun when the length of the shadow of a pole is J3 times the
height of the pole.

2. The angles of depression of two ships from the top of a light house are 45° and 30° towards east. If
the ships are 100 metres apart, find the height of the light house.
3. Two pillars of equal height are on either side of a roadway which is 30 metres wide. At a point on the
roadway between the pillars the elevations of the top of the pillars are 60° and 30°. Find the height
of the pillars and the position of the point.
4. A person standing on the bank of a river observes that the angle subtended by a tree on the
opposite bank is 60°; when he retires 14 metres from the bank he finds the angle to be 30°. Find the
height of the tree and the breadth of the river.
5. The upper part of a tree broken by the wind makes an angle of 60° with the ground and the distance
from the foot to the point where the top of the tree meets the ground is 20 metres. What was the
height of the tree?
Answers

1. a0 2 5031 m 3. # m
4. Height of tree = 7/3 m, width of river=7m 5. 20(2+3) m

SHIKHAR : ra Complex Vijaya Nagar, Chetakpuri Gwalior 0


NTUEE
EXERCISE - 1
Section (A) : Introduction to Trigonometry and trigonometric ratios

A1 If 5tan 6 = 4, then value of M is:


5sin6 + 2cos®
1 1 4 2
A) =3
(A) (BB) -3 ©)c) =5 D) =3
(D)

A-2. If 7sin o =24 cos a where 0° < a < 90°, then value of 14 tan o — 75 cos o.— 7 sec o is equal to :
(A)1 (B)2 ©)3 D)4

A-3. Iftan6=4, then Jtaine isequal to:


sin” 6
+sinBcosH
]

(A)0 B)22 ©) V2 D)1


A-4. If Ais an acute angle in a right AABC, right angled at B, then the value of sinA + cosA is :
(A) equal to one (B) greaterthanone (C) less than one (D) equal to two

Section (B) : Trigonometric angles


B-1. Asx increases from 0° to 90°, the value of cos x is :
(A) increases (B) decreases
(C) remains constant (D) increases, then decreases

- S Zsecitan’
B-2. Value of x from the equation xsin—cos?— = 6 3 4 is:
€ msecz%cosecg
(A)4 (B)6 ©)-2 Do

B-3. The area of a triangle is 12 sq. cm. Two sides are 6 cm and 12 cm. The included angle is 6, then
sinB=

(A)[E)1 ® |z1 ©|5]


1
O |3
1

B-4. IKfa+B=90°and a= 2, then cos? o + sin?p equals to :


1
(A) 2 (B0 ©1 D)2

B-5. GiventhatsinA= % and cos B =% ., where A and B are acute angles, then the value of A+ Bis :
<
(A) 30° (B) 45° (C)75° (D) 15°
Section (C) : Complementary angles

C1. Ifa+p=90°andsina= %,mensmpls:

A) =3
2 B) ——
22 2 3
(A) (8) 3 ©)C) =3 D) =2
(D)

C-2. The value of tan 5° tan 10°. tan 15°. tan 20° ... tan 85°, is :
A1 B)2 ©)3 (D) None of these

C-3. sin(60°+6) —cos (30° - 6) is equal to :


(A) 2 cosd (B) 2sin6 ©0 D)1

SHIKHAR : Indra Complex Vijaya Nagar, Chetakpuri Gwalior 0


SHIKHAR
MT-JEE
C-4. |If cos(a + B) =0, then sin(c. — B) can be reduced to :
(A) cos B (B) cos 2B (C)sin o (D) sin 2a.
Cc-5 IfsecA=oosecB=%,thenA*»Bisequalto:

(A) zero (8) 90° (C) <90° (D) > 90°

Section (D) : Trigonometric identities

D-1. Given3sinp + 5 cos p =5, then the value of (3 cos p — 5 sin p)?is equal to :

A) 9 B) =259 o 131 D

©o|=
(A) (®) ©) (D)

D-2. The value of [ (secA + tanA) (1 —sinA) | is equal to :


(A) tan’A (B) sin’A (C) cosA (D) sinA

D-3. If x =2sin%, y = 2 cos? + 1, then the value of x + y is :


1
(A2 ®)3 ©) 2 D)1

D-4. Value of (1 + tan 6 + sec6)(1 + cot6 —cosec ) is :


A1 B)-1 )2 (D)-4

D-5. The value of 5tan? — 5 sec?0 is :


A1 (®)-5 (O (D)5
ANSWER KEY
Section (A)
A1. (B) A2. (B) A3. (D) A4. (B)

Section (B)
B1. (B) B-2. (B) B-3. (D) B-4. (A) B-5. (C)

Section (C)

c4. (8) €2 (A c3. (© c4. (8) c5 (B)


Section (D)

D1. (A D-2. (© D3. (B) D-4. () D5 (B)

EXERCISE - 2
1. The length of the shadow of a pillar is % times of its height. The angle of depression of the sun is -

[Rajasthan NTSE Stage-1 2005]


(A) 30° (B)45° (C) 60° (D) 90°

2. From a point 200 metres away from the foot of a chimney, the angle of elevation of its top is 60°.
The height of the chimney is : [Rajasthan NTSE Stage-1 2006]

(A) 5043 m (B) 10043 m ©) % m (D) 2003 m


v

3. A tree is broken at a height of 5m from the ground and its top touches the ground at a distance of 12
m from the base of the free. The height of of the tree is : [Rajasthan NTSE Stage-1 2007]
(A)13m (B)5m (€)18m (D)17m

SHIKHAR : Indra Complex Vijaya Nagar, Chetakpuri Gw: r 0751-3551081


IT-JEE
4. The angles of elevations of the top of the tower from two points in the same straight line and at a
distance of 9 m. and 16 m. from the base of the tower are complementary. The height of the tower is
[Delhi NTSE Stage-1 2013]
(A) 18 m. (B)16 m. (C)10m. (D) 12m.
5. If the Angle of elevation of sun increases from 0° to 90° then the change in the length of shadow of
Tower will be — [Rajasthan NTSE Stage-1 2013]
(A) No changes in length of shadow (B) length of shadow increases
(C) length of shadow decreases (D) length of shadow will be zero

6. On the level ground, the angle of elevation of the top of a tower is 30°. On moving 20 metres nearer
to it the angle of elevation becomes 60°. The height of the tower is :[MP NTSE Stage - 1_2013]
(A)10m (B)15m (C)20m (D) 10/3m
7. The ratio of the height of a pillar and its shadow is 1 : +/3 . The angle of elevation of the sun is :
[MP NTSE Stage - 1_2013]
(A) 90° (B) 60° (C)45° (D) 30°
8. The angle of elevation of the top of a building from the foot of tower is 30° and the angle of elevation
of the top of the tower from the foot of the building is 60°. It the tower is 30 m high, then the height of
the building is [Rajasthan NTSE Stage-12014]
(A)30m (B)20m (C)15m (D)10m
9. The angle of elevation and the angle of depression are 30° and 60° respectively when seen from the
top of the first building to the top and base of the second building. If the distance between the bases
of two building is 12 m, then find the height of big building.
[Maharashtra NTSE Stage - 1_2014]
(A) 1643 m (8)124/3m (C) 1443 m (D)20+/3m
10. From the top of a 7 m high building, the angle of elevation of the top of a cable tower is 60° and the
angle of depression of its foot is 45°. The height of the tower in metre is
[Rajasthan NTSE Stage-1 2014]
A 7(B-1) ®)743 ©7+3 D73 +1)
1. A tower is 100+/3 m. high. The angle of elevation of its top from a point 100 m away from its foot is
........ [Bihar NTSE Stage-1 2014]
(A) 60° (B) 45° (C)30° (D) 22% °

12. The length of a ladder is exactly equal to the height of the wall it is leaning against. If the lower end
of the ladder is kept on a bench of height 3m and the bench is kept 9 m away from the wall, the
upper end of the ladder coincides with the top of the wall. The height of the wall is :
[Bihar NTSE Stage-1 2014]
(A)11m (B)12m (C)15m (D) 18 m

13. If a flagstaff of 6 metres high placed on the top of a tower throws a shadow of 23 metres along the
ground then the angle (in degrees) that the sun makes with the ground is
(Bihar NTSE Stage-1 2018)
(A) 60° (B) 30° (C)45° (D) None of these

ANSWER KEY
Ques.[ 1 2 3| 4 5 6|7 8| 9 |10) 11| 12] 13
Ans| C|D|C|D|D|D|ID|D|A|[D|[A]C]|A

NOTE: REFERENCE BOOKS:- RD Sharma (Class 10th), NCERT Textbook(Class 10th), NCERT Exemplar (Class 10)

Further reading:- MTG Mathematics(Class 10th), Pearson Mathematics(Class 10th), NTSE and NSEJS PYQs

SHIKHAR : Indra Complex Vijaya Nagar, Chetakpuri Gwalior 0751-3551081


SHIKHAR
TTJEE

CHEMISTRY
EXERCISE -1
1. The incorrect statement about metals is 10. When washing soda is treated with hydrochloric acid,
(a) metals have a lustre it gives off a colourless gas with lots of effervescence.
(b) metals are malleable Identify the gas.
(c) metals are non-ductile @@ Cco, (0, () CO @ ¢l
(d) metals form positive ions.
. Inwhich of the following test tubes, the gas on evolution
Which of the following processes is endothermic? extinguishes a burning matchstick?
(a) Burning of coal (b) Rusting
(¢) Freezing (d) Melting
The solution with the lowest concentration of H' ions
is
() pH=7 (b) pH=86
() pH=20 (d) pH=68 + H,0 CH;COOH + HCl
CH;COOH

Do ®
Number of electrons shared between carbon-carbon 1 I
atoms in ethene is

() 8
RO (b) 4
An element M forms an ion M™.
© 6 ) 2
The electronic
CH,COOH+Na,CO;
(a)
1
IIandIV
~ CH,COOH+NaHCO;
(b)
v
TandII
configuration of this ion is (2, 8). Which of the following (c) TandII (d) IIandIV
statements is correct?
. On moving from left to right in periodic table, which
(a) Misin group 16 and period 2 of the following properties is increases?
(b) Mis in group 17 and period 2 (a) Atomic radius (b) Valency
(c) Misingroup 17 and period 3 (c) Metallic character ~ (d) Atomic number
(d) Misin group 16 and period 3
-4 One of the following compound is not ionic in nature.
Heating the ore in the presence of excess supply of air This compound is
below its melting point is called (a) lithium chloride (b) ammonium chloride
(a) roasting (b) calcination (c) calcium chloride (d) carbon tetrachloride.
(c) smelting (d) liquation. 14. A student added a few drops of the
Which of the following is not a basic condition for universal indicator to a solution of /
combustion? dilute hydrochloric acid in the way s
(a) Fuel as shown here. E o opedl
universal
(b) Supporter of combustion He observed that the colour of the |5] (indicae
(c) Ignition temperature (d) Presence of nitrogen solution changes from colourless to dilute
(a) red (b) yellow & hydrochloric
A student took two test tubes, (A) and (B) containing
2 mL of dilute hydrochloric acid and added zinc granules
(c) violet (d) green. s
- The atomic numbers of four elements are given below:
to test tube (A) and solid sodium carbonate to test tube
(B) as shown below : Element w X X Z
Atomic number 5 12 15 20
Zinc .A sodium. _./
granules carbonate i3 ¥ Which of the following pairs of elements belong to the
same group of the periodic table?
(a) Wand X (b) WandY
i Dil. () XandZ (d) YandZ
hydrochloric
id 16. Four solutions labelled as A, B, C and D have pH values
) = ®
The correct observation would be 1,8, 3 and 13 respectively.
(a) rapid reaction in both the test tubes Which of the following statements about the given
(b) slow reaction in (A) and rapid reaction in (B) solutions is incorrect?
(c) rapid reaction in (A) but a slow reaction in (B) (a) Solution A has higher concentration of hydrogen
(d) no reaction in both the test tubes. ions than solution C.
Which of the following metals is protected even by a (b) Solution B has lower concentration of hydroxyl ions
than solution D.
layer of its oxide?
(a) Copper (b) Silver (c) Solutions A and B will turn red litmus solution blue.
(c) Iron (d) Aluminium (d) Solution A is highly acidic while solution B is
weakly basic.
SHIKHAR : Indra Complex ya Nagar, Chetakpuri Gwalior 0751-3551
IT-JEE
17. Phosphorus(V) chloride dissolves in water to form
phosphoric acid, H3PO, and hydrochloric acid. What
are the values of x, y and z in the balanced equation for
this reaction?
PClygy + xH,0(1)— y H3POyg + 2HCl) Platinum
x y z electrode electrode
(a) 2 1 4 Liquid X
[C) 2 5, Liquid ‘X’ is an aqueous solution of concentration
© 4 2 4 1 mol/dm®. Which substance would cause the bulb to
@ 4 1 5 glow the brightest?
18. (a) Ammonia (b) Ethanoic acid
Which of the following pairs of compounds of carbon
(c) Sugar (d) Sulphuric acid
will undergo combustion as well as addition reactions?
(a) CH,and C,H, (b) C,H,O and C;H,;0 7. Which of the following elements is not a metal?
() CH,0,and C;HeO (d) C,H,and C;H (a) Calcium (b) Copper
. Which of the following combination of elements belong (c) Potassium (d) Sulphur
to the same group? 28. When 1.0 mol of compound Q is burnt in excess oxygen,
(a) N,PAs (b) Li,Be, Al 2.0 mol of carbon dioxide and 2.0 mol of steam are
(c) Na, Mg, Al (@ 0o,s,Cl produced. Which of the following is the structural
formula of compound Q?
)
20. A student was given four unknown colourless samples H H
||
labelled A, B, C and D and asked to test their pH using (@ H—C—C—H (®) H—C—C=0
pH paper. He observed that the colour of pH paper
turned to light green, dark red, light orange and dark


blue with samples A, B, C, and D respectively. The correct
sequence of increasing order of the pH value of samples is
(a) A<B<C<D (b) A<D<C<B ()
i1
H—C—C—O—H
[
(d) H_?—?—(I:—O_H

21.
() C<B<A<D (d) B<C<A<D
On burning magnesium ribbon in air, it is observed that
o HOH B
29. Which of the following statements is correct about
(a) agreenish gas is obtained
elements '34 and }7B 7
(b) it burns brightly, giving golden colour
(a) A is more electronegative than B.
(c) it burns brightly, leaving a blue ash
(b) A forms a positive ion and B forms a negative ion.
(d) it burns brightly, leaving behind a white powder. (c) A and Bhave the same number of neutrons.
22. Consider the following table : (d) A and Bhave the same number of electrons.
Elements Electronic configuration 30. Observe the given figure and answer the question that
A 2,1 follows :
B 2,4
(o] 2,7
D 2,87
The formula of compound formed between elements
Aand D is China dish
(@) A,D (b)) AD () AD, (d) AD; Silver chloride
23, Which of the following is correct order of the reactivity
of the metals? State True (T) or False (F) for the following statements.
(@) Zn>Fe>Al>Cu (b) Al>Fe>Zn>Cu (i) Itisa photodecomposition reaction.
() Zn>Al>Fe>Cu (d) Al>Zn>Fe>Cu (ii) Silver chloride turns grey in sunlight to form silver
metal.
24, A student takes about 5 mL distilled water in four test
tubes marked P, Q, R and S. He dissolves sodium chloride
(i It is an exothermic reaction.
in P, potassium chloride in Q, calcium chloride in R (iv) Silver bromide also behaves in the same way and
used in black and white photography.
and magnesium chloride in S. After that he adds equal
amount of soap solution in each test tube. On shaking W G ) )
(@ T 1 T T
the solution of each test tubes he would observe a good
b) T F T F
amount of lather in the test tubes marked
() T P B T
(@) PandQ (b) QandR (c) RandS (d) PandS
@ T T F ir
25. In the reaction, Br, + 2I" — 2Br” + I,, the oxidising 31 Acids like lactic acid and uric acid which are obtained
agent is
usually from plants and animals, are known as
(a) Br, b) I (c) Br @ 1
(a) organic acids (b) inorganic acids
26. An experiment was carried out using the apparatus (c) oxyacids (d) hydracids.
shown below :
32. Arrange the following atoms in the order of increasing
atomic radius : N.B, C. §
(@ CSN,B (b) S,N,B,C
(c) C,BS,N (d) N,CBS

ya Nagar, Chetakpuri Gwalior 0


IT-JEE
33. Electrolytic reduction method is used in the extraction 41 . Which of the following pairs of compounds have similar
of
chemical properties?
(a) highly electronegative metals
(b) highl}"e]ectmposilivemela.ls lii III T

o @ nd-a H-ta
H H
34. The given diagram representsa reaction. l? }Ii fll }I{ Iil
() H—C—C—H H—C—C=C—H
b b
H H
| [
(© H—(li—(II:—O—H Hf?—C—O—H
(a) photodecomposition (b) electrolysis H H H
(c) displacement (d) thermal decomposition II{ H 1;1 le
35. Change of Na,COj3:10H,0 to Na,CO5-H,0 on exposure (d) H—C—é—Cl H—C—C—Cl
to air is called [ |
(a) efflorescence (b) effervescence HH i
() finorescence (d) luminescence. 42. Which one of the following types of ores canbe converted
into oxide by calcination?
36. The structural formulae of compounds Q and R are (a) Halide ores Carbonate ores
shown below : (c) Fluoride ores &)
(d) AllCa of these
H HH H\ H 43.
. )
Which of the following processes does not involve either
. )
H—C=C—C—C—H H N H oxidation or reduction?
}![ ]l[ H /\(4/ ~H (a) Formation of slaked lime from quick lime
H—C—H (b) Heating mercuric oxide
X e l\-I l{I \H (©) Formation of manganese chloride from manganese
Compound Q Compound R oxide . . X
Which statement about compounds Q and R is correct? {4); Foemstign/of slocifrom zinc blendg
(a) They belong to the same homologous series. 44. In a solution of pH = 5, more acid is added in order
(b) They have the same percentage composition by to reduce the pH = 2. The increase in hydrogen ions
mass. concentration is
(c) They have the same boiling point. (a) 100 times (b) 1000 times
(d) They have different general formulae. (c) 3times (d) 5 times.
37. The liquid non-metal is 45. Which of the following statements is not correct?
(a) mercury (b) bromine (2) A common functional group is present in different
(c) silicon (d) sulphur. members of a homologous series.
38. Which of the following elements will form an acidic (b) Two consecutive members of a homologous series
oxide? differ by a -CH, group.
(a) An element with atomic number 7. (c) The members of a homologous series can be
(b) An element with atomic number 3. represented by one general formula.
(c) An element with atomic number 12. (d) Different members of a homologous series have
(d) An element with atomic number 19. similar chemical properties.
39. Consider the following reaction: SOLUTION
o 1. (c) : Metals are ductile in nature.
2H,8(g + SOy = 2H,0¢, + 35 2. (d)
L . _— &4 3. (b): The solution with pH more than 7 are basic in
(1) and (II) are respectively nature hence solution with pH 8.6 has less concentration of
H* ions and more concentration of OH™ ions.
(a) oxidation reduction (b) reduction, oxidation
(c) oxidation, oxidation (d) reduction, reduction. 4. (b): Ethene contains a double bond between carbon-
40. Moist sodium bicarbonate was placed on a strip of pH carbon, formed by/ahiacing ofitwo'electrona palts.
paper. The colour of the strip HH H H
(a) turned blue (b) did not change He “H or H—(|: = (II —H
(c) turned green (d) turned light pink
5. (b): AsM has electronic configuration 2, 8 hence, M has
2, 7 configuration which belongs to fluorine (F). Therefore,
fluorine. i.e.. M lies in group 17 and period 2.
ya Nagar, Chetakpuri Gwalior 0751-3551
SHIKHAR
MT-JEE
6. (a) : Inroasting, ore is heated in excess of air to remove 28. (b): The molecular formula of the compound is C,H,0.
volatile impurities. Two carbon atoms will produce two moles of CO,. Four
7. (d) 8. (a)
hydrogen atoms will produce two moles of H,0.
(a) The molecular formula of the compound is C,Hg. Two
9. (d): Aluminium has a protective layer of oxide on it. carbon atoms will produce two moles of CO,. Six hydrogen
10. (a): Na,COj + 2HCl— 2NaCl + H,0 + CO,T atoms will produce three moles of H,0.
CO, is given out with brisk effervescence. (c) The molecular formula of the compound is C;HgO. Two
11. (a): Acetic acid reacts metal carbonates and metal carbon atoms will produce two moles of CO,. Six hydrogen
hydrogen carbonates to produce CO, gas which extinguishes atoms will produce three moles of H,0.
burning matchstick. (d) The molecular formula of the compound is C;HgO.
2CH,COOH + Na,CO;— 2CH;COONa + H,0 + CO, Three carbon atoms will produce three moles of CO,. Eight
hydrogen atoms will produce four moles of H,0.
CH;3;COOH + NaHCO;— CH;COONa + H,0 + CO,
29. (a)
12. (d)
30. (d): White silver chloride turns grey in sunlight due to
13. (d): Carbon tetrachloride the decomposition of silver chloride into silver and chlorine
14. (a) : When a few drops of universal indicator is added by light. It is an endothermic reaction.
to dilute hydrochloric acid it turns to red colour solution as Sunlight
2AgCl, —— 2Agy + Clyy
HClis acidic in nature. Silver chloride Silver (grey) Chlorine
Sunlight
15. (c) : Element Atomic no. Electronic configuration 2AgBr,) ——— 2Ag(y + Bryg
X 12 2,8,2 Silver bromide Silver Bromine
z 20 2,8,8,2
Both X and Z have 2 electrons in their outermost shell. 31. (a): Acids of living origin are called organic acids.
Hence, belong to same group. 32. (d): Atomic radius increases down the group while it
16. (c) : (a) Lower the pH of the solution, more acidic is the decreases from left to right in a period.
solution and higher is the concentration of H" ions. 33. (b): Highly electropositive metals are reduced by
Thus, solution A (pH = 1) has higher [H'] ions than solution electrolytic reduction method.
C(pH=3). 34. (d): It is a thermal decomposition reaction. When
(b) Higher the pH of the solution, more basic is the solution
mercury(II) oxide is heated strongly, it decomposes to give
and higher is the concentration of OH™ ions.
the elemental mercury and oxygen gas.
Thus, solution B (pH = 8) has lower [OH ] ions than solution
D(pH=13). 35. (a) : The process of loosing water of crystallisation when
(c) Solution A (pH = 1) is acidic which turns blue litmus exposed to air is called efflorescence.
solution red whereas solution B (pH = 8) is weakly basic 36. (b): Compounds Q and R have the same molecular
which will turn red litmus solution blue. formula, CsH,o. Hence they have the same percentage
(d) Solution A (pH = 1) is highly acidic while solution D composition by mass.
(pH = 13) is highly basic and solution B (pH = 8) is weakly Compound Q is an unsaturated hydrocarbon and compound
basic. R is a saturated hydrocarbon. They do not belong to the same
17. (d): PClsy) + 4H,00)— H3POy(aq) + SHCl(a) homologous series.
18. (d): C,H, and C;H, are unsaturated hydrocarbons. So, Compounds Q and R are different compounds. Hence, they
they can give both combustion and addition reactions. have different boiling points.
Compounds Q and R have the same general formula, C,H,,.
19. (a): N, P and As belongs to the same group.
37. (b): Bromine is a liquid non-metal.
20. (d) 38. (a) : The element with atomic number 7 has 5 electrons
2. (d): 2Mg + O, —> 2MgO in its outermost shell, hence it is a non-metal and will form
Magnesium Oxygen Magnesium oxide an acidic oxide. Rest are metals and form basic oxides.
(Air) (white powder) 9]
22. (b) 23. (@ Loss of oxygen - Reduction
24. (a) : Mg** and Ca®" ions make water hard and hard water 39. (b): 2H,S(+50, —> 2H;0()+3S,
does not form lather in soap. Thus, good amount of lather Loss of hydrogen - Oxidation 1
will be obtained in test tubes P and Q only. (I
25. @):Bn+2° 5 2Br+, 40. (a) : The colour of strip changed to blue because moist
sodium bicarbonate gives NaOH which is a strong base.
Gain of electrons 1
Reduction 41. (a) : They have the same functional group i.e., —Cl
The substance which undergoes reduction is an oxidising agent. group.
In option (b), the first compound has no functional group.
26. (d): As sulphuric acid is stronger acid so, the bulb will
The second compound has the C = C functional group.
glow the brightest.
27. (d): Sulphur is a non-metal. In option (c), the first compound has the functional group
[e]
Il
—OH. The second compound has the —C—O—H functional

ya Nagar, Chetakpuri Gwalior 0


SHIKHAR
MT-JEE
group. Students were asked to study the reaction between
In option (d), the first compound has the functional group —Cl. barium chloride and sodium sulphate. Four different
o]
reports of the experiment are given below. Choose the
The second compound has the functional group —&—Cl. correct one.
12. (b) Procedure Observation
(a) Mix powder of The colour of the
43. (a) : Formation of slaked lime from quick lime :
barium chloride and mixture becomes yellow.
CaO + H;0— Ca(OH),
This reaction does not involve the change in oxidation number sodium sulphate.
of any element. (b) Mix solution of White precipitate
barium chloride to is formed.
4. (b): [H*];=107; [H*]; =107
-2 the solution of
Thus, increase in [H'] = w—VS:IOOO times sodium sulphate.
(c) Add solution of Solution becomes
45. (b) : Two consecutive members of a homologous series
barium chloride to turbid.
differ by a —CH, group. sodium sulphate powder.
EXERCISE -2 (d) Add powder of The colour of the
1. The position of elements A and B are shown in the periodic barium chloride to mixture becomes bluish
table below. sodium sulphate solution. green.
In the Modern Periodic Table, calcium (Z = 20) is
surrounded by the elements with atomic numbers 12,
A
[T T T TTTTT 19,21 and 38. Which of the following will have physical
Which of the following statements is correct about the and chemical properties resembling calcium?
two elements? (a) 12,38 Mgl
iM
(a) A and B have the same electron structure. (b) 12,19,38 L
_IS. Ca ic_
(b) A and B have the same number of filled electron (c) 19,38
shells. (d) 12,19 iSr
(c) Aisametal and B is a non-metal. ‘What will be the pH value of a solution if salt of a strong
(d) Aisin period 2 and Bis in group IIL
base and weak acid undergoes hydrolysis?
2. For the given series of reactions, (@) pH=7 (b) pH>7 (c) pH<7 (d) pH=1
NaCl + X + Y + NH; — NaHCO; + NH,CI
Clean small pieces of
magnesium, zinc, aluminium,
Q——12Z
iron and copper by rubbing
Following statements are given :
them with a piece of sand
(i) The given reaction sequence is a Solvay's process.
paper. Take them in separate
(i) Xand Y are CO, and H,0 respectively.
test tubes. Add about 10 mL
(iii) Z is (NH,),CO;.
Identify the correct option. of dilute hydrochloric acid
(a) (i) and (ii) (b) only (iii) to each of them.
(c) (i) and (iii) (d) all statements are correct. Consider the following statements.
3. Pis produced by the action of chlorine on dry slaked (i) The rate of evolution of hydrogen gas bubbles is not
lime. Q is a non-corrosive base and used for faster same in all the test tubes.
cooking. (ii) The rate of formation of bubbles is the fastest in the
On heating R at 373 K, it becomes calcium sulphate case of magnesium.
hemihydrate. (iii) The reactivity decreases in the order :
The P, Q and R respectively are Mg >Zn > Al >Fe > Cu.
(a) CaOCl,, NaHCOs, gypsum (iv) In the case of copper, no bubbles are seen and the
(b) CaO, Na,COs, CaOCl, temperature also remains unchanged. This shows
(c) Ca(OH),, NaHCO;, CaSO, that copper does not react with dilute HCL.
(d) CaOCl,, Na,CO3, NH,Cl Identify the correct option.
4. A metal rod M was dipped in a coloured solution Y. (a) Only (i) and (iv) (b) Only (iii) and (iv)
After some time it was observed that the metal rod (c) Only (iii) (d) Only (i), (ii) and (iv)
starts dissolving in the Observe the given figure and identify the correct statements.
solution and the solution
starts fading in colour.
However, a coloured
precipitate Z was seen =
at the bottom of the beaker. M, Y and Z could be
(a) M=2Zn,Y=FeSO, Z=Fe
(b) M=Cu, Y=AL(SO,); Z=Al
(c) M=Ag Y=CuSO, Z=Cu
(d) M=Fe, Y=27nSO4 Z=2n
SHIKHAR
MT-JEE
white flame. By considering the following statements, identify the
(i) It is a displacement reaction. correct option.
(iii) Magnesium oxide is basic in nature and turns most 1. Xand Yare isomers.
red litmus paper blue. 2. Xand Y have the same percentage composition by mass.
(iv) It is an exothermic reaction. 3. Yis the isomer of alkane having formula, CsH,.
(a) (i) and (iii) (b) (i) and (iv) 4. Zhas the same boiling point as n-butane.
(0 (@), (iif) and (iv) (d) (i) and (iv) (a) 1,2and3 (b) 2and4
. Ethanoic acid was added to sodium bicarbonate solution (c) land4 (d) 1,3and4
and the gas evolved was tested with a burning splinter. 15. The given diagram shows the electron arrangement of
The correct observation is two elements, A and B.
(a) the gas burns with the pop sound and the flame gets
extinguished
(®) the gas does not burn but the splinter burns with a
pop sound
(©) the flame extinguishes and the gas does not burn By considering the following information, identify the
(d) the gas burns with a blue flame and the splinter correct option.
burns brightly. (i) Aisaperiod 3 element.
Four students performed the reactions of dil. hydrochloric (i) Bisa group I1I element.
11
(iii) Both A and B have strong metallic characteristics.
acid and solution of sodium hydroxide with zinc metal
and solid sodium carbonate separately. They reported (iv) Both A and B form positive ions.
the possible reaction by (¥') and no reaction by (x). (a) (ii) only (b) (i) and (ii) only
(c) (iii) and (iv) only (d) (i), (ii), (iii) and (iv)
Students | HCI HCl | NaOH | NaOH
+Zn |+Na,CO;| +Zn |+Na,CO;
16. Reaction between X and Y, forms compound Z. X loses
electron and Y gains electron. Which of the following
A v v v v
properties is not shown by Z?
B x x v v (a) Has high melting point.
G v v x x (b) Has low melting point.
D v v v x (c) Conducts electricity in molten state.
(d) Occurs as solid.
Which student has report the correct observation?
17. Consider the following statements.
@ A (b) B (© C @ D
(i) Dry hydrogen chloride gas turns dry blue litmus
. The ion of an element has 3 positive charge. Mass paper red.
number of the atom is 27 and the number of neutron (ii) Dry hydrochloric acid does not turn moist blue
is 14, what is the number of electrons in the ion? litmus paper red.
(@) 13 (b) 10 () 14 @ 16 (iii) IfdryHCl gas is passed through benzene, it will not
13. Identify the metal (X) and gas (¥) respectively in the turn dry blue litmus paper red.
figure. (iv) When HCl gas is passed through water, the solution
Metal X
Glass-wool turns dry blue litmus paper red.
soaked in Identify the incorrect option.
water
(a) (i) and (ii) (b) (i) and (iv)
(c) (i) and (iii) (d) (ii) and (iv)
Burner 18. Study the following steps.
Cug§ roasted in air * roasted without air m

Based on the above steps, identifyX and Y.


(a) Zinc, H, gas (b) Copper, CO,gas
(c) Aluminium, CO, gas (d) Platinum, H, gas X ¥
14. The structures of three hydrocarbons, X, Y and Z are (a) | Mixture of Cu and Mixture of Cu and
shown below: Cu0 S0,
H (b) | Mixture of Cu,0, Mixture of Cu and
H—$ ! ¢ ?—H,H—(II— —C—H; Cu,$ and SO, S0,
i
H—C—H H=¢—H g H—(—H
(c) | Mixture of CuO and
SO,
Mixture of Cu and
SO,
I o I (d) | Mixture of Cu and Mixture of CuO and

Ry
Hydrocarbon X Hydrocarbon Y CuO Cu$
There are four metals P, Q R and S. Identify them by

BH TflfH using the hints given below.


P forms basic oxide.
H—C—H Q forms amphoteric oxide.
Oxide of R dissolves in water to form alkali.
H S does not react with water at all.
Hydrocarbon Z
(@) P—Fe,Q—>Na,R—>K,S—Zn

SHIKHAR :
SHIKHAR
IT-JEE
(b) P—Zn,Q—>
AlLR— Na,S— Fe 6. (a): Because these elements belong to same group.
() P»Cu,Q—>Zn,R—-K
S—Pb 7. (b): Salt of a strong base and weak acid gives a basic
(d) P—K,Q—Cu,R—PbS—Na solution.
20. Which of the following flames is produced by a candle? 8. (d): The reactivity decreases in the order :
(2) Blue flame (b) non-luminous Mg > Al > Zn > Fe > Cu.
(c) light giving flame (d) yellow luminous flame
9. (c): Itis a combination reaction.
10. (c) : When ethanoic acid reacts with sodium bicarbonate.
Attempt any 5 out of 10.
CO, gas is evolved which does not promote flame.
21. From the given list of elements, how many pairs of The flame extinguishes and the gas does not burn.
elements form ionic bonds when they react?
Bromine, Carbon, Iron, Sodium, Sulphur 11. (d)

22. An element X belongs to group 14 and 2™ period of 12. (b): It is AI*" and has 13 protons and 10 electrons.
the periodic table. Its atomic number will be 13. (a): Zinc reacts with steam and produces zinc oxide
23. The pH of two solutions A and B are 2 and 4 respectively. and hydrogen gas. Platinum lies below hydrogen and is less
What is the ratio of H' ion concentration of the two reactive than hydrogen. Thus, it does not react with water.
solutions? 14. (a): Statement 1 : X and Y have the same molecular
24. Copper(II) oxide reacts with ammonia to give copper, formula, C;H,, but different structural formulae. X is a
water and nitrogen. branched chain alkane CH,?HCHZCHJ and Y is the straight
CuOy,) + NHj( — xCugy + yHO() + 2Ny
The value of (x + y + 2) is CH;3
chain alkane CH,CH,CH,CH,CHj.
25. CH,CH,0H —Ehaneicscd , oo mnound, Z Statement 2 : Since X and Y have the same molecular formula,
Number of hydrogens in compound, Z is they must have the same percentage composition by mass.
26. Among the following reactions, the number of reactions Statement 3 : Y is n-pentane. It is a structural isomer of
in which H,S$ acts as reducing agent is pentane having formula CsH,.
CuSO4+ H,S — CuS + H,S0,4 Statement 4 : Z is methylpropane. It is the isomer of butane.
Cd(NO3), + H,S — CdS + 2HNO, Isomers have different boiling points.
2FeCl, + H,S — 2FeCl, + 2HCl + §
15. (b): Element A belongs to period 3 and element B
27. Number of valence electrons in an atom of an element belongs to group IIL.
which belongs to group 18 and first period in the
periodic table is 16. (b): X —> X' +e;Y+e — Y
The compound Z formed by X* and Y~ ions will be ionic
28. Among the following oxides, number of basic oxides
and ionic compounds have high melting point.
are
Na,0, MgO, CO,, §0,, K;0, NO,, CuO 17. (a) : Dry hydrogen chloride gas does not turn blue litmus
29. The number of structural isomers possible for the red as dry hydrogen chloride gas does not give H* ions.
molecular formula CgH, 4 are Acids produce H"(,,) ions in solution which are responsible
for acidic properties of acids. Hydrochloric acid turns blue
30. The number of gaseous product(s) obtained when
litmus red.
electricity is passed through an aqueous solution of
brine is 18. (c) : Cu,S on roasting forms copper oxide and sulphur
dioxide gas escapes out. Roasting of copper oxide and left
SOLUTION
over Cu,$ in the absence of air forms copper metal and SO,
1. (c): Ais a metal and B is a non-metal. gas is evolved.
2. (a): NaCl+ H,0 + CO, + NH;— NaHCO; + NH,Cl 19. (c) : CuQ is basic in nature, ZnO is amphoteric in nature.
w0 m Alvfizo, -co, Oxide of K dissolves in water to form alkali.
Na;CO5-10H,0 <20 Na,c0, K0, + HyO() —> 2KOH 4
Pb does not react with water at all.
Washing soda Sodium carbonate
@ @ Thus, P, Q, R and S are Cu, Zn, K and Pb respectively.
3. (a): Ca(OH),+ Cly—»CaOCl, +H,0 20. (d)
Slaked lime Chlorine Bleaching 21. (4): Given elements :
‘powder
® Bromine (Br) - Non-metal
Q, baking soda (NaHCO;) is a mild non-corrosive base Carbon (C) - Non-metal
and it is added for faster cooking. On heating R, gypsum Iron (Fe) - Metal
(CaSO,4-2H,0) at 373 K, it loses water molecules and becomes Sodium (Na) - Metal
calcium sulphate hemihydrate (CaSO, 4512 Lh,0). Sulphur (S) - Non-metal
A metal forms ionic bond with a non-metal.
4. (a): Znlies above Fe in the reactivity series, hence it is
more reactive than Fe and displaces Fe from FeSO, solution.
Thus, ionic bonds are formed by :
Zng, + FeSO, o —> ZnSOyq + Fey Bromine and sodium (NaBr), Sulphur and iron (FeS), Sodium
o “ @ and sulphur (Na,S) and Iron and bromine (FeBr;)
5. (b): BaCly(gy) + Na;SO,(s —> BaSOy, + 2NaClig,
(White ppt.)

SHIKHAR : Indra Complex ya Nagar, Chetakpuri Gwalior 0


SHIKHAR
SUNTAR
IT-JEE
22. (6): Group 14 shows that it has 4 electrons in outermost 28. (4) : Na,0, MgO, K;0, CuO are basic oxides.
shell and 2™ period shows it has 2 shells hence, the electronic
29. (5): CH; — CH, — CH, — CH, — CH,— CH,
configuration of the element will be 2, 4 or atomic number n-Hexane
of the element will be 6 (carbon). CH; — C|H — CH, — CH, — CH;
23. (100): pH = 2 means [H'] =107 M
pH = 4 means [H'] = 10 M CH,
2-Methylpentane
As 1072 > 107, solution A is more acidic than solution B. CH, — CH, — CH — CH, — CHj
[H']in solution A 107 M - |
107 CH,
[H']insolution B 107 M 3-Methylpentane
i.e., [H"] in solution A is 100 times more than that in solution B. CH,
24. (7): 3CuO)+ 2NH3— 3Cugy + 3H,00) + Nogg
|
CH;—C—CH,—CH; CH;— CH— CH— CH,
25. (8): . |
CH,CH;0H + CH;COOH —2— CH,COOCH,CH, CHj CH; CH;
2,2-Dimethylbutane 2,3-Dimethylbutane
Ethanol Ethanoic Bthyl ethanoate
acid @ 30. (2): Electrolysis
2NaCly,y + 2H,0( Elec
26. (1) : 2FeCl3 + HS — 2FeCl; + 2HCl + S trolysis
2NaOH(yy) + ClypT + HyyT
27. (2) : The element is helium. It has 2 valence electrons.
(At anode) (At cathode)

NOTE: REFERENCE BOOKS :- NCERT Textbook(Class 10th), NCERT Exemplar Textbook (Class10th)

Further reading(Optional):- S. Chand (Class 10th), MTG Chemistry(Class 10th), Pearson Chemistry (10th)
NTSE and NSEJS PYQs

PHYSICS (CHAPTER - 1

1. Multiple Choice Questions Q.6. If an object is placed symmetrically between two


plane mirrors , inclined at an angle of 72°, then total
number of images formed is :
Q.1. Multiple images are formed by a thick plane mirror.
Which of the following is the brightest of all the (a) 5 ® 4
images? © 2 (d) infinite
(a) first (b) second Q.7. A plane mirror produces a magnification of :
(c) third (d) fourth (@ -1 ®) +1
Q.2. Aplane mirror is placed vertically facing due north. () zero (@) between 0 and infinity
Anarrow pointing north-east is kept in front of the Q.8. Adiceis placed with one of its edges parallel to the
mirror. In which direction will the arrow point in principal axis between the principal focus and the
this image? centre of curvature of a concave mirror. Then the
(a) north-east (b) south-east final image has the shape of :
(c) south-west (d) north-west. (a) cube
Q.3. When a plane mirror is rotated through a certain (b) rectangular parallelepiped
angle, the reflected ray turns through twice as much (c) barrel shape (@) spherical
and the size of the image : Q.9. Two adjacent walls and the ceiling of a rectangular
(a) is doubled (b) is halved room are mirror surfaced. The number of images
(c) becomes infinite (d) remains same of himself that an observer sees is :
Q.4. The image formed by a convex mirror is only one- @ 3 ®9 ©6 @8
third of the size of the object. If the focal length of Q.10.Image formed by a convex spherical mirror is :
the mirror is 12 cm, the image formed will be : (a) virtual (b) real
(@) 8 cm behind the mirror (c) enlarged (d) inverted
(b) 10 cm behind the mirror Q.11.Which of the following can make a parallel beam
of light when light from a point source is incident
(c) 8cmin front of the mirror
onit? [NCERT Exemplar]
(d) 10 cm in front of the mirror
(@) Concave mirror as well as convex lens
Q.5. Abeam of light incident on a plane mirror forms a
real image on reflection. The incident beam is :
(b) Convex mirror as well as concave lens
(@) parallel (b) convergent (c) Two plane mirrors placed at 90° to each other
(@) Concave mirror as well as concave lens
(¢) divergent (d) not certain

SHIKHAR : Indra Complex ya Nagar, Chetakpui


IT-JEE
Q.12.A 10 mm long awl pin is placed vertically in front (@ real, inverted and of the same size as that of the
of a concave mirror. A 5 mm long image of the awl object
pin is formed at 30 cm in front of the mirror. The Q.20.Two plane mirrors are at right angles to each other.
focal length of this mirroris: [NCERT Exemplar] A man stands between them and combs his hair
with the right hand. In how many images will he
(@) —30em &) -20cm
be seen using his left hand ?
(©) —40cm (@) -60 cm
(@) none ®1
Q.13. Which of the following statements is true ?
@ 2 @ 3
[NCERT Exemplar]
(a) A convex lens has 4 dioptre power having a ANSWERS,
focal length 0.25 m 1L®» 20 3. @ 4 @ 5. (b)
(b) A convex lens has - 4 dioptre power having a 6. 7. 0 8 ® 9 (@ 10 @
focal length 0.25 m 11. @ 12. () 13. (@ 14. @ 15. ()
(c) A concave lens has 4 dioptre power having a 16. () 17. (@ 18. () 19. (@ 20. (h)
focal length 0.25 m
(d) A concave lens has —4 dioptre power having a 2. Assertions and Reasoning Type
focal length 0.25 m
Questions
Q.14. Magnification produced by a rear view mirror fitted
in vehicles : [NCERT Exemplar]
Directions : In the following questions, two statements
are given-one labelled Assertion (A) and the other labelled
(a) is less than one
Reason (R). Select the correct answer to these questions
(b) is more than one from the codes (a), (b), (c) and (d) as given below :
(0) is equal to one
(@) Both (A) and (R) are true and (R) is correct
(@) can be more than or less than one depending
upon the position of the object in front of it explanation of the assertion.

Q.15.Rays from Sun converge at a point 15 cm in front of ® Both (A) and (R) are true but (R) is not the correct
aconcave mirror. Where should an object be placed explanation of the assertion.
so that size of its image is equal to the size of the (©) (A)is true but (R) is false.
object ? [NCERT Exemplar] () (A)is false but (R) is true.
(@) 15 cm in front of the mirror
Q1 Assertion (A) : A convex mirror is used as a
(b) 30 cm in front of the mirror driver’s mirror.
(c) between 15 cm and 30 cm in front of the mirror
Reason (R) : Images formed in a convex mirror
(d) more than 30 cm in front of the mirror are diminished.
Q.16.A full length image of a distant tall building can
definitely be seen by using: [NCERT Exemplar] Assertion (A) : In going from a denser to a rarer
medium, a ray of light bends away from normal.
(a) aconcave mirror (b) a convex mirror
() aplane mirror Reason (R) : This occurs because light travels faster
(d) both concave as well as plane mirror in a rarer medium than in a denser medium.
Q.17.You are given water, mustard oil, glycerine and Q.3. Assertion (A) : For total internal reflection, angle
kerosene. In which of these media a ray of light of incidence in rarer medium must be greater
incident obliquely at same angle would bend the than critical angle for the pair of media in
most ? [NCERT Exemplar] contact.
(@) Kerosene (b) Water
(c) Mustard oil (d) Glycerine 1
Reason (R): n = =,
G where the symbols have
Q.18.A child is standing in front of a magic mirror. She
finds the image of her head bigger, the middle
portion of her body of the same size and that of the
their standard meaning.
legs smaller. The following is the order of Q4. Assertion (A) : When a convex lens of power
combinations for the magic mirror from the top. + 2D is held in contact with a concave lens of
[NCERT Exemplar] power — 4 D, the combination has a focal length
(a) Plane, convex and concave of-05m.
(b) Convex, concave and plane Reason (R): P =P, + P,
(c) Concave, plane and convex Q.5. Assertion (A) : Refractive index of a medium
(d) Convex, plane and concave decreases with the rise in temperature of the
Q.19. An object is placed at a distance of 40 cm in front of medium.
a concave mirror of focal length 20 cm. The image Reason (R) : Refractive index of a medium varies
produced is : directly as the density of medium and density
(a) virtual and inverted decreases with the rise in temperature.
(b) real and erect
(c) real, inverted and of the same size as that of the ANSWERS,
object
Lb 2@ 3. @ 4. ® 5. B

SHIKHAR : ra Complex Vijaya Nagar, Chetakpuri Gwalior 0


SHIKHAR
ST
IT-JEE

The focal length of a concave mirror is 30 cm. Find the 1 v I 6.7


position of the object in front of the mirror so that the Aleo, "EoTTuT
T T2
image is three times the size
of the object. [CBSE-2016] 6.7 x4.5
Therefore, 1= 1—)(2 =+25cm
Sol. (i) If the image is real and inverted.
I=+25cm
-3ie,v=3u,f=-30cm The positive sign indicates that image formed is
erect and virtual.
Using mirror formula %+ —= %, we have Q4. A convex mirror used on an automobile has 3 m radius
of curvature. If a bus is located at 5m from this mirror,
Iofl_pds mBtlatinl, gl 1 find the position, nature and size of the image.
u 3u -30 3u -30 3u -30 [AI-2020]
u=-40cm
GivenR=+3m,f=+15m,
Thus, the object should be placed at a distance
Object distance u =—5m,
of 40 cm in front of the mirror.
Image distancev=7?,
In this case, v =3u = — 120 cm.
Image size i’ =?
The image will be formed at a distance of
120 cm in front of the mirror. Using mirror formula A 1+l
f u v
(ii) If the image is erect and virtual.
3 111
1 1,5+15
m=-—=3o0rv=-3u % f u 15 -5 75
u
75
orv= " 1.15m
Using the relation L ,we have
u v f
The image is 1.15 m behind the mirror.
1.1 1 3-1 1
—+——=——or —— =——oru=-20cm K :
u Bu -30 3u =30 Magnification, m = == D el +0.23
u -5
Thus, the object should be placed at a distance
The image is virtual, upright and smaller in size
of 20 cm in front of the mirror and the image
than the object by a factor of 0.23 (about
formed.
one-fourth).
Also,v=-3u=-3x-20=+60cm Q5. An object is placed 50 cm from a lens that produces a
Q2. An object is placed 15 cm from a convex mirror of radius virtual image at a distance of 10 cm in front of the lens.
of curvature 60 cm. Find the position of image and its Calculate the focal length of the lens.
magnification. [CBSE-2016] Givenu= -50cm, v=-10cm,f=?
Sol. Given u =-15cm, R = + 60 cm or f = + 30 cm,
v=?andm="?
Using the formula -1 -1
v u f
. % 1114
Usmgmm’orformula;+;=? 1111 .1 _-4
f o u -10750 50
1.1 1_1 1 _1+2_3 -50
YT TN BT % %0 orf=T=—12.5cm
The negative value shows that the lens is concave.
~v=10cm ?
Q.6. The power of a lens is 2.5 dioptre. What is the focal
v 10 2
length and the type of lens ? [Delhi-2015]
me == 573
Thus, the size of image is double than the size of Sol. Power of alensis P = Y
the object and is virtual. f
Since power = 2.5 dioptre
Q3. A 4.5 cm needle is placed 12 cm away from a convex 1
mirror of focal length 15 cm. Give the location of the 25= ? orf= 5 m=04m=40cm
image and its magnification. [CBSE-2016]
The focal length of the lens is 40 cm. Since the focal
Sol. Given thatO=45cm, u=-12cm,f=+15cm
length is positive, therefore, the lens is convex lens.
Using the mirror formula 1 + =L, A convex lens of focal length 10 cm is placed
at a distance
of 12 cm from a wall. How far from the lens should an
object be placed so as to form its real image on the
or wall ?
Sol. Givenf=+10cm, o=+ 12 cm (since the image is to
Therefore, v=6.7cm be formed on the wall which is on the other side as
The positive sign indicates that the image formed that of the object), u=?
is on the other side of the mirror.

SHIKHAR : Indra Complex Vijaya Nagar, Chetakpuri Gw:


SHIKHAR
MT-JEE
Using the lens formula L-1-1 wehave wehave I=-8cm
Thus, the image is real and mvertgd
v u

1.1.1_1 1 _10-12_-2 -1 Q2. Find the position, nature and size of the tmage af an
W v f 12 10 120 120 60 object 3 cm high placed at a distance x?‘
or u = — 60 cm. Thus, the object is placed at a concave mirror of focal length 18 cm. [AISS! 2004’
distance of 60 cm in front of the lens. Sol. GivenO=3cm,u=-9cm, f= -18cm, v=?,
4 k=2
The refractive index of water is 3 and for glass is % % 2 1. 9.1
Using the mirror formula —=—+— ,wehave
with respect to water. What is the refractive index of fiR 0
glass with respect to water ?
2. Aol L 11
Sol. Given g, = =and,,us=%
v f u -18 -9 18
Therefore, v=+18 cm
aMy
Weknow that, ;g = =t , therefore, we have Theimageis formed 18 cm behind the mirror. Thus,
the image is virtual and erect.

Now, m= L
Q9. h
A 5 cm tall object is placed perpendicular to the
principal axis of a convex lens of focal length Therefore, we have W=
20 cm. The distance of the object from the lens is
30 cm. Find the (i) position, (ii) nature and (iii) sizeof Thus, the size of the image is 6 cm.
the image formed. [CBSE-2014, Q3. A point object is placed at a distance of 12 cm from a
GivenO=5cm,u=-30cm, f =+20cm,v=2and convex lens on its principal axis. Its image is formed on
I=? the other side of the lens at a distance of 18 cm from the
Using lens formula -1 -11 wehave lens. Find the focal length of the lens. Is the 1ma§
v u
magnified? Justify your answer. BSE-2011]
111 1,11 Sol. Givenu=-12cm,v=18cm,f=?
v f u 20 -30 60
Therefore,v = 60 cm Using the formula 1-1-1 ,wehave
I o ik a v u
A]so,m=6=;,them_fole.m=g 30 Solving, L - i
we have 18 -12 f
I=-10cm
Solving for f, we have f=7.2cm
The image is real, inverted and magnified.
Q.10. A 10 cm tall object is placed perpendicular to the v_ 18 _ 3
principal axis of a convex lens of focal length
30 cm. The distance of the object from the lens is Thus, image is magnified.
20 cm. Find the (i) position, (ii) nature and (iii) size Q14 (4) Twolenses have power of (i) + 2 D (ii) — 4 D. What
the image formed. [CBSE-2014,
is the nature and focal length of each lens ?
GivenO=10cm,
u=-20cm, f = +30cm, v="?and
1=? (b) An object is kept at a distance of 100 cm for a lens
of power — 4D. Calculate image distance.
[CBSE-2012]
Ans. (@) (i) Convex lens of focal length + 50 cm.
(i) Concave lens of focal length 25 cm.
(b) GivenP= -4D,u=100cm,v="?
1 1
Also, m = l = =, therefore, m =
I —60 Now, f= P-4 =-025m=-25cm
10 =20 . 3 1 .12
Solving, we have Using the relation = =—,wehave
I=+30cm u

The image is virtual, erect and magnified. 1 1 1 1 1


QL = gt—=—
An object 4 cm high is placed at a distance of 2 f u 25 -100
27 cm in front of a convex lens of focal length
18 cm. Find the position, nature and szzea the image = T
formed. SE-201 100 20
v=-20cm
Sol. GivenO=4cm, u= -27cm, f=+ 18cm,v=7 Q15 (g A concave mirror produces three times enlarged
andI=? image of an object placed at 10 cm in front of it.
Using lens formula l—l =1 ,we have Calculate the focal length of the mirror.
v ou f
(b) Show the formation of the image with the help of a
1t .11 ray diagram when object is placed 6 cm away from
v f u 18 -27 54 the pole of a convex mirror. [CBSE-2012]
Therefore,
v = + 54 cm
Ans. (a) Givenm=-3 (Realimage), u =—10cm, f=?
A]so,m:é:%,thmfm’e,m:%: 57 Solving, 4
IT-JEE
Therefore, -3 = % orv=3u=-30cm CHAPTER -2 (HUMAN EYE)
Using the mirror formula, we have
1_1
——— = 1 1 s Multiple Choice Questions
f =30 -10 75
Q1. A person cannot see distinctly objects kept beyond
Therefore, f=—7.5cm
2 m. This defect can be corrected by using a lens of
(b) The ray diagram is as shown :
power : [NCERT Exemplar]
@) +05D b -05D
@ +02D @ -02D
Q2. Cinematography makes use of:
(a) accommodation
(b) persistence of vision
(c) least distance of distinct vision

Q16. 4p object 1 cm tall is placed at a distance of 15 cm from


(d) bifocal lens system
a concave mirror of focal length 10 cm. Find the position, Q3. The human eye forms the image of an object at its :
size and nature of the image formed. [CBS -2013f (a) cornea (b) pupil
(c) iris (d) retina
Sol. GivenO=1cm,u=-15cm,f=-10cm,v=? . The change in focal length of an eye lens to focus
the image of objects at varying distances is done by
Using the mirror formula % - + A ,wehave
u v the action of the:
1 -3+2 -1 (@) pupil (b) retina
or “150 30 30 (c) ciliary muscles (@ blind spot
Therefore, v=-30cm
. The colour of light which is deviated the least by a
The image is real and inverted.
prism in the spectrum of white light is:
1 v
Now, m=5="3 (a) red (b) green
(c) violet (@) yellow
o rem Lix0me T0 wd
u -15 Q6. Which of the following is correct?
=-2cm @) Myie > Myettow > Mgreen
Q17. A 2.0 cm tall object is placed perpendicular to the ® Ayetow > Agreen > Moiue
principal axis of a convex lens of focal length 10 cm.
© Ageltow > Mplue > Agreen
The distance of the object from the lens is 15 cm. Find @ Agreen > Moiue > Ayetlow
the position, nature and size of the image éorms. In case of human eye, when light rays enter the eye,
Q7.
[CBSE-2013]
most of the refraction occurs at the:
Sol. GivenO=20cm,f=+10cm,u=-15cm,0=?,
(a) crystalline lens
1=?
1 (b) outer surface of the cornea
Using the lens formula 1L L ,wehave
v ou f (c) iris
O g 60 O ot 3L (d) pupil
v fiu 1015~ 30 30 orv=30cm Q.8. Reddish appearance of the sun at sunrise or sunset
The image is formed on the other side of the lens at is significantly due to:
30 cm from it. (a) dispersion of light
1 (b) scattering of light
e el Lu X O -15 2.0 m i (c) total internal reflection of light
(d) reflection of light from the earth
The negative sign shows that the image is real and
Q9. Identify the correct statement regarding the
inverted. propagation of light of different colours of white light
in air.
(a) red light moves fastest,
(b) blue light moves faster than green light.
(c) all the colours of the white light move with the
same speed.
(d) yellow light moves with the mean speed as that
of the red and the violet light.

SHIKHAR : Indra Complex Vijaya Nagar, Chetakpuri Gwalior 0


IT-JEE
Q.10. Identify the phenomena of light involved in the atmosphere
formation of a rainbow. Q.18. Which of the following statements is correct
(a) reflection, refraction and dispersion. \ regarding the propagation of light of different colours
(b) refraction, dispersion and total internal of white light in air ? [NCERT Exemplar]
reflection. (@) Red light moves fastest
() dispersion, scattering and total internal (b) Blue light moves faster than green light
reflection. (c) All the colours of the white light move with the
(d) refraction, dispersion and internal reflection. same speed
Q.11. The focal length of the eye lens increases when eye (@ Yellow light moves with the mean speed as that
muscles : [NCERT Exemplar] of the red and the violet light
(a) are relaxed and lens becomes thicker. Q.19. Which of the following phenomena contributes
(b) contract and lens becomes thicker. significantly to the reddish appearance of the sun at
(c) are relaxed and lens becomes thinner. ( sunrise or sunset ? [NCERT Exemplar]
(@ contract and lens becomes thinner. (a) Dispersion of light
Q.12. At noon the sun appears white as: [NCERT Exemplar] (b) Scattering of light
(a) light is least scattered. (c) Total internal reflection of light
(b) all the colours of the white light are scattered
(d) Reflection of light from the earth
away.
Q.20. The bluish colour of water in deep sea is due to :
(c) blue colour is scattered the most.
(d) red colour is scattered the most. [NCERT Exemplar]
Q.13. A prism ABC, with base BC, is placed in four different (a) the presence of algae and other plants found in
orientations as shown. A narrow beam of white light ‘water
is incident on the prism as shown. In which of the ‘ (b) reflection of sky in water
following cases, after dispersion, the third colour (c) scattering of light
from the top corresponds to the colour of the sky? (d) absorption of light by the sea
[NCERT Exemplar]
& . A c c ANSWERS,

LB 2® 3@ 40 50
6® 7@ 8® 90 100
1@ 120 B @ W@ 150
A B c B B
6. 0 17.(0 18. © 19..0) 20. @
) (@ (i) ()
@@ @G (G (@ G SOLVED ILLUSTRATIONS
Q.14. A person cannot see distinctly objects kept beyond
2 m. This defect can be corrected by using a lens of Q1 A person with a defective eye-vision is unable to see the
power: objects nearer than 1.5 m. He wants to read books at a
(a) -02D (b -05D distance of 30 cm. Find the nature, focal length and
(@ +02D @ +05D power of the lens he needs in his spectacles.
[CBSE-2016]
Q.15.Which of the following phenomena of light are Sol. This person suffers from the defect of hyperme-
involved in the formation of a rainbow ?
tropia.
[NCERT Exemplar] Forhimu=-30cm, v=-15m =-150cm
(a) Reflection, refraction and dispersion Therefore, focal length of corrective lens to be
(b) Refraction, dispersion and total internal used by him is
reflection 1 11
(c) Refraction, dispersion and internal reflection or f=375cm
(d) Dispersion, scattering and total internal
fovou ~150
reflection. The positive sign shows that the lens needed is
a convex lens of focal length 37.5 cm.
Q.16. Twinkling of stars is due to atmospheric :
Hence, power of lens needed
[NCERT Exemplar]
(a) dispersion of light by water droplets 1 _ 100
(b) refraction of light by different layers of varying T f 375
refractive indices Q2. The
far point of a myopic person is 80 cm in front of the
(c) scattering of light by dust particles eyes. What is the nature and power of the lens required
to enable him to see very distant objects distinctly ?
(d) internal reflection of light by clouds.
Q.17. The clear sky appears blue because : [CBSE-2016]
[NCERT Exemplar] Sol. For a myopic eye, the correcting lens is concave.
(a) blue light gets absorbed in the atmosphere Distance of far point, x =80 cm,P=?
(b) ultraviolet radiations are absorbed in the For viewing distant objects, focal length of
atmosphere corrective lens,
(c) violet and blue lights get scattered more than =-x=-80cm
lights of all other colours by the atmosphere Using the relation
(d) light of all other colours is scattered more than
the violet and blue colour lights by the

Indra Complex ya Nagar, Chetakpuri Gwalior 0


IT-JEE
Q3. “Far point of a myopic person is 40 cm. What fype of CHAPTER - 3 (ELECTRICITY)
lens should he wear so as to see the distant objects
clearly? Calculate the focal length and the power of the
lens he should use. [AI-2015] 1. Multiple Choice Questions
Sol. u= e (infinity), v=-40 cm, f=?
Using the lens formula, we have Q1 Electrical resistivity of a given metallic wire
depends upon : [NCERT Exemplar]
(a) its length (b) its thickness
(c) its shape (d) nature of the material
Q2. If a wire of resistance R is melted and recast to
half of its length, then new resistance of the wire
will be :
(@ R/4 (&) R/2
©R @ 2R
Q3. A current of 1 A is drawn by a filament of an
electric bulb. Number of electrons passing
Q4. A myopic person has been using spectacles of power through a cross-section of the filament in 16
~1.00 D for clear vision. During old age he also needs seconds would be roughly : [NCERT Exemplar]
to use separate reading glasses of power + 2.00 D. (a) 1020 (b) 106
Explain what may have happened to his eyesight?
Sol. ‘When he was using only lenses for myopic defect,
(o) 1018 @ 102
his near point was normal i.e., equal to 25 cm. His Q4. If R; and R, are the resistances of filaments of a
far off point was 100 cm according to the power of 400 W and a 200 W lamp, designed to operate on
lens he was using. His eye lens had the normal the same voltage,
then :
power of accommodation. Due to old age, his
ciliary muscles became weaker and he did not
@R, =2R, ®) Ry =2R,
have a normal power of accommodation. So his (@R, =4R, @R, =R,
near point of vision also increased and reached In a circuit containing two unequal resistors con-
up to 50 cm as shown below and he needed a lens nected in parallel :
of 2.00 D power for that purpose.
(a) the current is same in both the resistors.
P=+200D
(b) the current is large in the larger resistance.
o f=p=i=05m (c) the voltage is same across both the resistors.
=50cm, u=-25cm (d) the voltage drop is larger across the larger
Using the lens formula, we have resistance.
Q. The equivalent resistance in series combination
oit W el WS R O is:
v T sT 0 (a) smaller than the largest resistance.
or v = 50 cm: (b) larger than the largest resistance.
Thus, the near point of this person is 50 cm away (c) smaller than the smallest resistance.
from his eyes.
(d) larger than the smallest resistance.
The far point of a myopic person is 150 cm in front of
Q7. A house is fitted with 5 tubes each of 40 W. If
the eye. Calculate the focal length and the power of a all the tubes are lighted for 20 hours and if the
lens required to enablehim to see distant objects clearly. cost of electrical unit is ¥ 0.50, the total cost of
[CBSE-2020] electricity consumed is :
Given u = (infinity), v=-150cm,f=? @20 (H)4 (I @22
Q8. Of the two bulbs in a house, one glows brighter
than the other. Which of the two has a larger
resistance?
(a) the dim bulb
Y T (b) the brighter bulb
f7A150
or f=-150cm=-15m (c) both have the same resistance
Hence, power of lens to be used is (d) becomes four times
You are given a resistance wire of length 50 cm
P= 1 =L:_0_67D and a battery of negligible resistance. In which
15 of the following cases is the largest amount of
A person needs a lens of power — 0.5 dioptre for
correcting his distant vision. For correcting his near heat generated:
vision he needs a lens of power + 1.5 dioptre. What is (@) when only half of the wire is connected to
the focal length of the lens required for correcting his the battery.
(i) distant vision, (ii) near vision? [CBSE-2011] (b) when the wire is divided into four equal parts
Sol. For distant vision and all the four parts are connected in
f=1/P=1/-05=-05m parallel.
For near visions (c) when the wire is divided into two equal parts
f=1/P=1/+15=0.67m and both the parts are connected in parallel.

SHIKHAR : Indra Complex Vijaya Nagar, Chetakpuri Gwalior 0


SHIKHAR
MT-JEE
(d) when the wire is connected to the battery (c) Brightness of bulb B will be more than that
directly. of A
Q0. Which of the following represents voltage ? (d) Brightness of bulb C will be less than that of B
[NCERT Exemplar] . In an electrical circuit two resistors of 2 Q and
4 Q respectively are connected in series toa 6 V
@ Work done battery. The heat dissipated by the 4 Q resistor
@ Currentx Time in 5 s will be : [NCERT Exemplar]
(b) Work done x Charge @ 5] ® 10]
C© Work done x Time ©20] @ 30]
Current Q9. An electric kettle consumes 1 kW of electric
power when operated at 220 V. A fuse wire of
(d) Work done x Charge x Time what rating must be used for it ?
. A cylindrical conductor of length ! and uniform
[NCERT Exemplar]
area of cross-section A has resistance R. Another
conductor of length 2! and resistance R of the @ 1A ® 2A
same material has area of cross-section: ©4A @ 5A
[NCERT Exemplar] Q.20. Unit of electric power may also be expressed as:
@) A/2 ) 3A/2 [NCERT Exemplar]
() 2A ) 3A (a) volt ampere (b) kilowatt hour
Q12. What are the essential requirements of a heater (c) watt second (d) joule second
filament?
ANSWERS.
(a) high resistivity, low melting point
(b) low resistivity, low melting point L@ 2 @ 3. @ 4. b 5 ©
(c) high resistivity, high melting point 6. (b) 7. d) 8. (@ 9. ( 10. (@
(d) low resistivity, high melting point. 1L () 12. () 13. (@ 14. (© 15. (0
16.(c) 17. (0 18. © 19. (@ 20. (@)
Q13. Two wires of the same metal have the same
length, but cross-sections are in the ratio 3:1. SOLVED IL! RATIONS
They are joined in series. The resistance of the
thicker wire is 10 ohm. The total resistance of Q.1. An electric iron has a rating of 750 W; 200 V.
the combination will be : Calculate:
(@) 40 ohm (b) 40/3 ohm (i) the current required.
() 5/2 ohm (d) 100 ohm. (i ) the resistance of its heating element.
. You are given three equal resistors. The number
of resistances which can be obtained by joining (iii) energy consumed by the iron in 2 hours.
them in series and in parallel grouping is : [CBSE - 2016]
@) two () three i 750 W
Sol. () Current required =75557 =3.75 A
(c) four @ six
Q.15. If the current I through a resistor is increased by 200V
100% (assume that temperature remains (i) Resistance = 375A " 533V
unchanged), the increase in power dissipated
(iii) Energy consumed =750 W x 2h=1500
W h
will be : [NCERT Exemplar] =15kWh
(a) 100% (b) 200%
Q2 A piece of wire is redrawn, without change in volume
(c) 300% (d) 400%
so that its radius is halved. Compare the new
Q.16. The resistivity does not change if :
resistance with the original resistance. [CBSE - 2016]
[NCERT Exemplar]
Sol. Let length of original wire is ! and radius is r.
(a) the material is changed
(b) the temperature is changed SoR=p nr
1
(c) the shape of the resistor is changed
(d) both material and temperature are changed Original volume V= nr?l
Qa7. In an electrical circuit three incandescent bulbs When its radius halved it becomes % and let
A, B and C of rating 40 W, 60 W and 100W
respectively are connected in parallel to an length of redrawn wire becomes I".
electric source. Which of the following is likely ThenR’=4p L
to happen regarding their brightness ?
[NCERT Exemplar]
and volume V, = "(5)r I d
(@) Brightness of all the bulbs will be the same
(b) Brightness of bulb A will be the maximum Volume is same or V; =V,

SHIKHAR : Indra Complex Vijaya Nagar, Chetakpuri Gw:


SHIKHAR
NTUEE
£ of cross- section 2A i.e.,
or.ml=m (E) 1
L
or I'=4] . L,l:; and A, =2A
Substitute value in R’ = p 4 7 L
-k, Using equation = DX , we have
()
. 4lx4 . Ix16 [
R,=
Ri=p-_ ; Ri=p—rs A,
Dividing, we have
R’=16 xR
By Lyt L2 A1
So resistance increased by 16 times. R A, L 20 L 4
Q3. (a) How much current will an electric bulb draw Since the old resistance is 4 Q, therefore, the new
from a 220 V source, if the resistance of the bulb resistance becomes one-fourth of the previous
filament is 1200 ohm? [CBSE - 2015] value. Hence, R, = 1 ohm.
(b) How much current will an electric heater coil Q7. An electric bulb is connected to a 220 V generator.
draw from a 220 V source, if the resistance of
The current is 0.50 A. What is the power of the bulb?
the heater coil is 100 ohm?
. Given V=220V,I=050A
Sol. (a) Given V=220V,R=1200Q,1=?
We know thatP = VI
Using the expression V = IR, we have
=220x050= 110 W
1=220V/ 1200 2 =0.18 A.
(b) Given V=220V,R=100Q. .~ How much work is done in moving a charge of
From expression V = IR, we have 2 coulomb from a point at 118 volt to a point at
128 volt?
1=220V/100Q=22A.
Q4. The potential difference between the terminals of an
Sol. Given §=2C,V,=118V
electric heater is 60 V when it draws a current of
and V,=128V,W=2
4 A from the source. What current will the heater
We know that V = %, therefore,
draw, if the potential difference is increased to 120 V?
Sol. We are given, potential difference V = 60 V, W =q(V,-V;) =2(128 - 118) =20]
current =4 A. - Two resistors are connected in series as shown in the
diagram. (i) What is the current through the 5 ohm
According to Ohm’s law, V = IR i.e., current is resistor? (ii) What is the current through R?
directly proportional to the applied potential (iii) What is the value of R? and (iv) What is the
difference. value of V?
Therefore, when the potential difference is
increased to 120 V i.e., made double the current
will also become double. Therefore, current
through the heater becomes 8 A.
. Resistance of a metal wire of length 1 m is 26 2 at
20°C. If the diameter of the wire is 0.3 mm, what will
be the resistivity of the metal at that temperature? V volt
-
Sol. Given R=26Q, 1;
D=03mm=3x10%m, Sol. Given
L=1m Potential difference across 5 ohm resistor = 10 V
Using the expression Potential difference across R ohm resistor = 6 V.
RaD? Value of resistance Ry =50hm,[=?, R=?and V=7
_&BA. By Ohm'’s law, the current through resistor of
L 4L 5ohm is
26x3.14x(3x107)
S
=184x10°Qm Since the two resistors are connected in
This material is manganese. series, therefore, the current through
resistor R is also 2 ampere.
Q.6. A 4 Qresistance wire is doubled on itself. Calculate Therefore, value of R can be obtained as
the new resistance of the wire. [CBSE - 2015] follows :
Sol. GivenR=4Q.
When a wire is doubled on it, its length would
@ R=2=%_30m
11, 71270m

become half and area of cross-section would (iii) Since the resistors are in series, therefore, net
double. That is, a wire of length L and area of resistance of the circuit is
cross-section A becomes of length L/2 and area R=R;+R;=5+3=8Q

SHIKHAR : Indra Complex Vijaya Nagar, Chetakpuri Gw:


SHIKHAR
MT-JEE
Hence, by the expression V = IR, we have (ii) the total current flowing in the circuit.
V=2x8=16V
2.10. When two resistors of resistances R; and R, are [CBSE-2005]
connected in parallel, the net resistance is 3 . When
connected in series, its value is 16 $2. Calculate the 100 200
values of Ry and Ry. (AI-2016) 5Q 25 Q
Sol. When R; and R; are connected in parallel, net 1
resistance (Rp) is given by
1 metpe—
1 1 =
12v
Rp Ry Ry
Sol. () Resistors of value 10 Q and 20 Q are
Therefore, we have connected in series, therefore, their
RiRy equivalent resistance is
Rp=R,4R, =3 (1)
Rs=10+20=30Q
When R, and R, are connected in series, net
Similarly, resistors of value 5 Q and 25 Q
resistance (Rg) is given by
are in series, therefore, their equivalent
Rg=R; +R, =16 .(2) resistance is
From equations (1) and (2), we have =5+25=30Q
RiR,
16 =3 or RiR,=48 -(3) Now, Rg and R, are in parallel, therefore,
we have
Using equations (2) and (3), we have
Ry(16-R,) =48 Roz RsRs, 3030 _ %00
Since (R; + R, = 16, or R, = 16 - R,) PTRe+Rs, 30430 60
or 16R,- R} =48
. =150
(ii) The total current is given by
Solving for R;, we have Ry = 4 Q or 12 Q.
Therefore, the resistances of two resistors are = l:E =08 A

4Qand 12 Q. Rp
Q.11. An electric bulb is rated at 200 V-100 W. What is Q.14. Two identical resistors, each of resistance 2 £, are
its resistance? Five such bulbs burn for 4 hours. connected in turn : (i) in series, and (ii) in parallel
What is the electrical energy consumed? Calculate to a battery of 12 V. Calculate the ratio of power
the cost, if the rate is 50 paise unit. (AI-2020) consumed in the two cases. [CBSE-2012]
Sol. GivenV=200V,P=100W,n=5E=? Sol. Given Ri=2Q,R,=2Q, V=12V
(i) When resistors are connected in series, then
v2 3 Rg=2+2=4Q.
(a) We know that P= — orR = 3 therefore,
resistance of each bulb is Therefore, power consumed

=Xv2 = (200 =40 Q V2 (12) 2


P 100 400 Ps=q = 4 =3%W
(b) Electrical energy consumed by one bulb in 4
(ii) When resistors are connected in parallel,
hours then
=100x4=400Wh
Rp= RiR2 _2X2 0
Energy consumed by 5 bulbs in 4 hours
Ri+R, 2+2
=400x5=2000Wh=2kWh
Therefore, power consumed
(c) Cost of consumed electricity
=2x05=%1 v (12
=—= =14 W
Q:12. A torch bulb is rated 5 V and 500 mA. Calculate its PPRp 1
(i) power (ii) resistance and (iii) energy consumed P 144
when it is lighted for 4 hours. [CBSE-2005] Hence, ratio Bs = 36 =4
Sol. Given V=5V, 1=500mA =05A,P=?R="?
Q.15. In the given circuit, calculate (i) total resistance of the
and W=7
circuit, and (ii) current shown by the ammeter.
() P=VI=5x05=25W Ry=50 (CBSE-2012]
i) R=V/I=5/05=10Q
(i) W=Pxt=25x4h
=30 Rz=2Q
= 2.5% (4x3600 s) = 36000 J
Q.13. Jfa 12 V battery is connected to the arrangement of
resistances given below, calculate :
(i) the total effective resistance of the arrangement and 25V
Sol. () Since R, and R, are in series, their resultant
resistance,

SHIKHAR : Indra Complex Vijaya Nagar, Chetakpuri Gw:


IT-JEE
Rs=R,+R,=3 +2=5Q Also, Rss and Ryg are in series, therefore,
Further, Rs and Rj are in parallel, their wehave Rsgs=1+1=20hm
resultant resistance is given by
Now, Ryp34 and Rsgyg are in parallel, hence, net
resistance of the circuit is
1 1 1
(ii) Also, :R_—S:IA = +
R Ry Regrs
Q.16. In a household, 5 tube lights of 40 W each are used Hence, equivalent resistance = 1 ohm.
for 5 hours and an electric press of 500 W for 4 hour
everyday. Calculate the total energy consumed by the Q.24. A hot plate of an electric oven, connected to a 200 V
line. It has two resistance coils A and B each of the
tube lights and press in a month of 30 days.
30 which may be used separately, in series or in
[CBSE-2013]
Sol. Electrical energy consumed by 5 tube lights of parallel. Find the value of the current required in each
40 W each for 5 hours per day of the three cases. [CBSE - 2012]
=5x40x5 Sol. Given V=220V,
=1000Wh=1kWh
Electrical energy consumed by electric press of
Resistance of coil A (R,) = Resistance of coil B
500 W for 4 hours per day (Rg) =30Q
=500 x4 =2000Wh=2kWh (i) When coil A or B is used separately, then
Total electrical energy consumed per day I1=V/R=220/30=733 A
=1kWh+2kWh=3kWh (ii) When the coils are connected in series, the
Total electrical energy consumed in a month of total resistance is
30 days
E=3kWhx30=90kWh
Rg =Ry + Ry =30 + 30 = 60 ohm
Hence, current
Q.17. A wire is 1.0 m long, 0.2 mm in diameter and has a
resistance of 10 €. Calculate the resistivity of its
Is=V/Rg=220
/60 =3.67 A
(i) When the coils are connected in parallel, the
material. [CBSE-2011] total resistance is
Sol. Given ~ L=10m,D=02mm
4 _ 1,1 1 1 1
or r=01mm=0.1x10%m, Rp Ry, Ry 30 30 15
R=10Q,p=? or Rp=15Q
Using the relation Hence, current is
_eL_pL I=V/R=220/15=14.67 A
A 1"2 Q.25. An electric heater rated 1200 W operates 2 hour per
3)2 day. Find the cost of the energy required to operate
RA 10)(3.14)((0.1)(10 )
or p=f = — it for 30 days at ¥ 5.00 per unit. ~ [CBSE - 2013]
1 Sol. Power of the electric heater = 1200 W
=314x107Qm
Number of hours per day heater is used = 2
Q.23. Find the equivalent resistance across the two ends A
Energy consumed per day = 1200 x 2
and B of this circuit. [CBSE - 2012]
=2400W
Total energy consumed in 30 days = 2400 x 30
=72000 W
=72kW
=72 units
Cost of energy = Total no. of units x Rate
=72x5
=3360
Sol. From the diagram, we find that the following
NOTE: REFERENCE BOOKS :- NCERT Textbook(Class
pairs of resistors are in parallel. 10th), NCERT Exemplar Textbook (Class10th)
R, and Ry, Ry and Ry, Rs and Rg, and R; and Rg.
Therefore, we have Further reading(Optional):- S. Chand (Class 10th),
1 1 1 1 1 MTG Physics(Class 10th), Pearson Physics (10th)
R117R71+R1 =z +E—'lohm NTSE and NSEJS PYQs

or Ryp=10hm
Similarly, Rg4 = 1 ohm, Rsg = 1 ohm and Rpg = 1
ohm.
Now, Ry, and Ry, are in series, therefore,
Ryz34=1+1=20hm
Also, Rgs and Ryg are in series, therefore,

SHIKHAR : Indra Complex Vijaya Nagar, Chetakpuri Gwalior 0


SHIKHAR
NT-JEE
SAMPLE ENTRANCE TEST SET:
PART MATHS

SECTION - I | (Single Correct Choice Type)

This Section contains 15 Single choice questions. Each question has four choices (A), (B), (C) and (D) out
of which ONLY ONE is correct.
Marking Scheme:
You will be awarded 3 marks for correct answer, -1 for wrong answer and zero if Question is left un-
attempted.
1. The value of
22,33, 4114 423, 5715
T1075 595 435,613 is equal to
¢ 6 @ 18
(@ 10 ®) 1
2. Ifthe polynomial f(x) = ax® +bx—c is divisible by the polynomial g(x) = x* +bx +c, thenab =
*
(fa) 1 b) —1
c
4
() -1 G
c

3. In the given figure, AB = AC, ZA =48 and £ ACD = 18°. BC equal to -

(@ AC (*b) CD
(¢) BD d) AB
4. In figure PQ is a chord of a circle and PT is the tangent at P such that
ZQPT =60°. Then £ PRQ s equal to
(a) 135
(b) 1500
() 120°
) 110°
5. Two circles of radii 20 cm and 37 cm intersect in A and B. If O1 and Oz
are their centers and AB= 24 cm, then the distance 0102 is equal to
(@) 44cm (*b) 51cm
(c) 405cm (d) 45cm

6. Consider the following statements :


I. The centroid of an acute angled triangle lies in the interior of the triangle.
IIl. The orthocenter of an acute angled triangle lies in the interior of the triangle
IIl. The circumcenter of an acute angled triangle lies in the interior of the triangle
IV. All the above
(a) land Il (b) neither land Il (c) llandlll (d*) AILL L NIl are correct

Directions : (7 to 8) : The lengths of two parallel chords of a circle are 6 cm and 8 cm. The smaller chord is at a
distance of 4 cm from the centre.
7. Theradius of the circle is
(@ 10cm (*b) 5cm (c) 3cm (d) None of these

8. Thedistance of the other chord from the centre is


(*Ya) 3cm (b) 6cm (c) 4cm (d) None of these
Directions : (9 to 10) :

SHIKHAR : Indra Complex Vijaya Nagar, Chetakpuri Gw:


In the above given figure, a, circular arc of radius 6 cm has been drawn with vertex O of an equilateral
triangle OAB of side 12 cm as centre.
9. The area of the sector AOB’ is
(Ya) 6rcm? (b) mem? () 6cm? (d) (6+m)ecm?

10. The area of the complete shaded region is


(a) 651.522cm? (*b) 156.552 cm? (c) 165.552 cm? (d) 561.552cm?
11. If x = 4sin(90° — 6) cot (90° — 6) and cosec6=2 , then x =
(@ o () 1 () -1 (d) 2
12. A ladder is included to a wall making an angle of 30° with it. A man is ascending the ladder at the rate of 2
metres/second. How fast is he approaching the wall?
(@ 2m/s (b) 1.5m/is (*¢) 1mis (d) 25mis

13. If the segment joining the points ( a, b) and (c, d) subtends a right angle at the origin, then
(@) ac-bd=0 (b*) ac+bd=0 (c) ab+cd=0 (d) ab-cd=0

14. A large basket of fruit contains 3 orange, 2 apples and 5 bananas. If a piece of fruit is chosen at random, what is
the probability of getting an orange or a banana?
. 4 1 7
(a) 3 ©) - © 75 (d) None of these

15. The mean of 11 number is 35. If the mean of first 6 numbers is 32 and that of last 6 number is 37, then the 6"
number is equal to
(@) 28 (*b) 29 (c) 31 (d) None of these

PART - II : APTITUDE

SECTION - I | (Single Correct Choice Type)

This Section contains 15 Single choice questions. Each question has four choices (), (B), (C) and (D) out
of which ONLY ONE is correct.
Marking Scheme:
You will be awarded 3 marks for correct answer, -1 for wrong answer and zero if Question is left un-
attempted.

Directions (16) : There is a number series. In each case, numbers follow a pattern. One place is left blank in each
series. Only one of the four alternatives is correct
16. 8, 20,45, 96, 199, ?
(*a) 406 (b) 420 (c) 408 (d) 256
Directions (17) : In each of the following questions various terms of an alphabet series are given with one or more
terms missing as shown by (?). Choose the missing terms out of the given alternatives.
17. A2C, D7D, I22E, P67F, ?
(a*) Y202G (b) Y201G (c) Z201G (d) Z202G
18. What was the day on 15t January 1901?
(a) Monday (b) Wednesday (c) Sunday (*d) Tuesday

19. In a clock, the angle between the hour hand and minute hand at 5 hour 10 minutes, is
(a*) 95° (b) 105° (c) 1100 (d) 1150
20. The priest told the devotee. “The temple bell is rung at regular intervals of 45 minutes. The last bell was rung 5
minutes ago. The next bell is due to the rung at 7: 45 am”. At what time did the priest give this information to the
devotee?
(@) 6:50am (b) 6:55am (c) 6:15am (d*) 7:05am

Directions (21 to 22) : Study the following information carefully and answer the questions given below:
Eight friends Q, R, S, T, V, W,Y and Z are sitting around a circular table facing the center. There
are three males and five females in the group of friends. No two males are immediate neighbors
of each other.
(i) V sits second to the right of his wife.
(ii) S sits third to the right of V.
(iii) W sits second to the right of her husband Z, Z is not an immediate neighbour of V's wife.
(iv) T is @a male and Y is not an immediate neighbor of V.
(v) R sits second to the right of Q.
SHIKHAR : Indra Complex Vijaya Nagar, Chetakpuri Gw:
NT-JEE
21. What is the position of T with respect to Z?
(@) Second to the left (b) Immediately to the right
(c) Third to the left (*d) Third to the right

22. Who amongst the following sits exactly between V and Y?


@) Q (b) W () R d T
Directions : (23 to 24) : In the figure, the circle stands for employed , the square stands for hard working, the triangle
stands for rural and the rectangle stands for intelligent. Study the figure carefully and answer
the question that follows

23. Non-rural, employed, hard working and intelligent people are indicated by region
@ 8 ") 9 (c) 10 @@ M
24. Unemployed rural hard working and intelligent people are indicated by region
(@ 1 (b) 2 () 3 (d) 4
25. Neelam , who is Deepak’s daughter, Says to Deepika. “Your mother Rekha is the younger sister of my father
who is the third child of Ramlal. “How is Ramlal related to Deepika?
(a) Uncle (b) Father
(c*) Matimal Grandfather (d) Father —in-law

26.

8| 7]10]12
13 | 1215 | 17
109 )7 |14
(a) 12 (b) 10 () 21 d) 25

@ &
27. Two positions of a dice are shown below. Identify the number at the bottom when the top is “3"?

(@ 1 (b) 4 ) 5 d) 2
Directions : (28) : The following question are based on the information given below
A cuboid shaped wooden block has 4 cm length, 3 cm breadth and 5 cm height.
Two sides measuring 5 cm x4 cm are coloured in red.
Two faces measuring 4 cm x 3 cm are coloured in blue.
Two faces measuring 5 cm x 3 cm are coloured in green.
Now the block is divided into small cubes of side 1 cm each
28. How many small cubes will have three faces coloured?
(a) 14 (*b) 8 (c) 10 d 12
29. Count the number of triangles and squares in the given figure
(a) 36 triangles, 7 squares
(b) 38triangles, 9 squares
(*c) 40 triangles, 7 squares
(d) 42 triangles, 9 squares

30. Find the number of triangles in the given figure.


SHIKHAR
SUNTAR
IT-JEE
PA III : SCIENCE

SECTION -1 (Single Correct Choice Type)

This Section contains 15 Single choice questions. Each question has four choices (), (B), (C) and (D) out
of which ONLY ONE is correct.
Marking Scheme:
You will be awarded 3 marks for correct answer, -1 for wrong answer and zero if Question is left un-
attempted.

31. A particle experiences constant acceleration for 20 s after starting from rest. If it travels a distance X4, in the first
10s and distance Xz, in the remaining 10s, then which of the following is true?
(@) Xi=2X2 (b) Xi=X2 (c) X1=3X2 (*d) None of these

32. A vehicle moving along a linear road takes the road it will go in 4t.
Speed

2V ey
;
Hr—
|H | Ry
00— A Time
Since the velocity-time graph of the vehicle is as shown in the figure, how many t took the vehicle to the first half
of theroad?
3 5
(@ 1 o) () 2 @ 5
33. When a net force acts on an object, the object will be accelerated in the direction of the force with an
acceleration proportional to the
(*a) Force on the object (b) Velocity of the object
(c) Mass of the object (d) Inertia of the object

34. The ratio between masses of two planets is 2 : 3 and the ratio between their radii is 3 : 2. Then the ratio
between acceleration due to gravity on these two planets is
(a*) 8:27 (b) 27:8 (c) 9:4 d) 3:5
35. A luminous object is placed 60 cm from surface of a convex mirror and a plane mirror is set so that virtual
images formed in two mirrors coincide. If plane mirror is at a distance of 40 cm from object, then focal length of
convex mirror is
(@ 25cm (*b) 30cm (c) 40cm (d) 60cm

36. What is the resistance between P and Q?


3
(a)a 2
—Q 0 20
4
®)b 3
=Q W
(*c) 16, 20 20
3
(d) Infinity 73 o

37. A dynamo develops 1A at 8V. The energy which it generates in one second is
(@) 84J (b) 4J () 2J (d) zero
38. Five equal resistors when connected in series dissipated 5 W power. If they are connected in parallel, the power
dissipated will be
(@) 125W (b) 96W (c) 68W d) 32w
39. Which of the following would weigh the highest?
(@) 0.2 mole of sucrose (C12H22013) (b) 2 moles of CO2
(c*) 2 mole of CaCOs (d) 10 moles of H.0

40. The pH of the gastric juices released during digestion is


(*a) lessthan7 (b) more than7 (c) equalto7 (d) equalto0

41. Which of the following is incorrect about the ??CI’ ion?


(a) lts mass number is 35 (b) The number of protons is 17
(*¢) The number of electrons is 16 (d) The number of neutrons is 18

SHIKHAR : Indra Complex Vijaya Nagar, Chetakpuri Gw:


NT-JEE
42. On moving from left to right across a period in the modern periodic table, the number of valance electrons
(*a) increase (b) decrease
(c) firstincrease then decreases (d) remains same

43. The number of covalent bonds present in ethane molecule is


@ 6 *b) 7 ) 8 ) 5
44. The name of the compound CH; —CH, -CHO is
(a*) Propanal (b) propanone (c) propanol (d) propenol

45. Copper articles develop a green coating when exposed to humid atmosphere due to the formation of
(@) copper hydroxide (b*) basic copper carbonate
(c) copper oxide (d) both (a) and (b)

PAPER-II
PART MATHS

(Subjective Type Questions)

This Section contains 9 Subjective Type Questions. Write solution for each question along with final
answer within the space provided.
Marking Scheme:
You will be awarded 5 marks for correct solution with correct answer and zero in all other cases. There is
no step marking.
46. The LCM of two numbers is 14 times their HCF. The sum of LCM and HCF is 600. If one number is 280 then
find the other number.
Solution: 80

47. A person standing on the bank of a river observes that the angle of elevation of the top of the tree standing on
the opposite bank is 60°. When he moves 30 m away from the bank, he finds the angle of elevation to be 30°.
Find the height of the tree and the width of the river.
Solution: height of the tree 15«/5 m and width of the river 15 m

48. Inthe given figure, if DE || BCand AD : DB =5 : 4, then find M


ar(AABC)
A

D, E

B C
Solution: 25
81

49. If A(—4, 8), B(-4, -2), C(0, -5) and D(0, 5) are the vertices of a quadrilateral ABCD, find its area.
Solution: 40 sq units

50. In the given figure, tangents PQ and PR are drawn from an external point P to a circle with centre O, such that
£ RPQ = 30°. A chord RS is drawn parallel to the tangent PQ. Find £ RQS.

S \R

307
o)

Solution: 30°

SHIKHAR : Indra Complex Vijaya Nagar, Chetakpuri Gw:


SHIKHAR
NT-JEE
51. In the given figure, AB is a chord of a circle ,with centre O, such that AB = 24 cm and radius of circle is 15 cm.
Tangents at A and B intersect each other at P. Find the length of PA.

Solution: 20cm

52. Infigure, AB and CD are two diameters of a circle (with centre O) perpendicular to each other and OD is the
diameter of the smaller circle. If OA = 7 cm, find the area of the shaded region.

Solution: 66.5 cm?

53. The height of a cone is 30 cm. From its topside a small cone is cut by a plane parallel to its base. If volume of
smaller cone is 1/27 of the given cone, then at what height it is cut from its base?
Solution: 20

54. The probability of selecting a red ball at random from a jar that contains only red, blue and orange balls is Y.
The probability of selecting a blue ball at random from the same jar is 1/3. If the jar contains 10 orange balls,
find the total number of balls in the jar.
Solution: 24

PART - II : SCIENCE

(Subjective Type)
This Section contains 6 Subjective Type Questions. Write solution for each question along with final
answer within the space provided.
Marking Scheme:
You will be awarded 5 marks for correct solution with correct answer and zero in all other cases. There is
no step marking.
55. A metro train stops at two stations P and Q which are 2 km apart. It accelerates uniformly from P at 1 ms~' for
20 seconds and maintains a constant speed for a time before decelerating uniformly to rest at Q. If the
deceleration is 0.5 ms2, find the time for which the train is traveling at a constant speed.
Solution: 70s

56. Find the value of ratio of Lwhere fk and fi represents the focal length of two convex rays shown in diagram.
3
The dots are equally placed.

Solution: 2
3

SHIKHAR : Indra Complex ya Nagar, Chetakpuri Gw:


SHIKHAR
NT-JEE
57. The three resistance A, B and C have values 3R,6Rand R respectively. When some potential difference is
applied across the network, the thermal powers dissipated by A, B and C are in what ratio?
3R
A R
— 6R —/\/\/\/—-C

B
Solution: 4:2:3

58. The total numbers of moles of atoms present in 250 g of magnesium nitride having molecular mass equal to 100
u?
Solution: 10

59. Count the number of single and double bonds in Butanal. Write its formula and structure and calculate its
molecular mass.
Solution: Do it yourself

60. Write the chemical formula of all the following compounds


Ethyl propanoate, Sulphurous acid, gypsum, cuprous oxide, oxalic acid
Solution: Do it yourself

SAMPLE ENTRANCE TE! SET-B


PART MATHS

SECTION - I |(Single Correct Choice Type)

This Section contains 15 Single choice questions. Each question has four choices (), (B), (C) and (D) out
of which ONLY ONE is correct.
Marking Scheme:
You will be awarded 3 marks for correct answer, -1 for wrong answer and zero if Question is left un-
attempted.
n. 520 330
1. The value of Mis equal to
3" %320 x 2%
@ 1 (b) 37" () 37 (*d) .3"

2. What is the condition for one root of the quadratic equation ax? +bx+c = Oto be twice the other?
(a) b?=4ac (*b) 2b% =9ac (6) c?=4a+b? (d) c*=9%a-b?
3. In the adjoining figure.
A
3,
E

DE || BC and all measurements are in centimeters. The length of AE is


(@ 2cm (*b) 2.25cm (c) 35cm (d) 4cm

4. In figure, if O is the centre of a circle, PQ is a chord and the tangent PR at P makes an angle of 50° with PQ,
then ZPOQis equal to

(*a) 1000 (b) 800 () 900 ) 75


SHIKHAR
SUNTAR
IT-JEE
5. If a regular hexagon is inscribed in a circle of radius r, then its perimeter is
(@) 3r (*b) 6r (c) or d) 12r
Consider the following statements:
|. The orthocenter of a right angled triangle is the vertex containing right angle.
1l. The circumcenter of a right angled triangle is the mid point of its hypotenuse.
11l. The centroid of a right angled triangle lies in the interior of the triangle.
IV. All the above.
Which of the statement(s) given above is/are correct?
(a) land Il (b) neither I nor I (c) llandlll (*d) Al I, Il are correct

Directions (7 to 8) : In the adjoining figure, DE || BCand AD :BD=4:3


4

D)

B c
7. The value of AD is

(@) % (b*) % (c) % (d) None of these

area(ADEF)
area(ADEC)
(@) %1 (b) % (*c) % (d) None of these

Directions (9 to 10) : A test tube is in the form of a right circular cylinder, surmounted by a cone. The diameter of the
cylinder is 24 cm. The height of the cylindrical portion is 11 m, while the vertex of the cone is 16 cm
above the ground.

The curved surface area of the cylindrical portion is


(a) (2467 )m? (*b) (264m)m? (c) (426 mm? d) (462mm?
10. The slant height of the cone is
(@ 31m (b) 15m (c) 16m (d*) None of these

xcosec?30° sec?45°
1. = tan260° — tan? 30° ,then x =
8cos? 45 sin” 60°
(a) 1 (b) -1 (€ 2 @ o
12. A 25 m ladder is placed against a vertical wall of a building. The foot of the ladder is 7 m from the base of the
building. If the top of the ladder slips 4 m, then the foot of the ladder will slide
(@ 5m (b*) 8m () 9m (d) 15m

13. If A=(a?2a) and B=[—- -2 and S =(1,0), then ——+—1-=


a?' a SA " SB
(@ 2 b)) = (e) 1 [CH R
14. In a shipment of 100 televisions, 6 are defective. If a person buys two televisions from that shipment, what is the
probability that both are defective?
3 9 1
a — ) — €) — d None of these
@ 100 ® 2500 (o 330 @

15. In the frequency distribution, the mid value of a class is 10 and the width of the class is 6. The lower limit of the
class is
(@ 4 (b) 7 (c) 16 d) 13

SHIKHAR : Indra Complex Vijaya Nagar, Chetakpuri Gw:


SHIKHAR
SUNTAR
IT-JEE
PART - 1I : APTITUDE
SECTION - I [Single Correct Choice Type)
This Section contains 15 Single choice questions. Each question has four choices (), (B), (C) and (D) out
of which ONLY ONE is correct.
Marking Scheme:
You will be awarded 3 marks for correct answer, -1 for wrong answer and zero if Question is left un-
attempted.

Directions (16) : There is a number series. In each case, numbers follow a pattern. One place is left blank in each
series. Only one of the four alternatives is correct
16. 9,19,3575, 143,?
(a) 287 (b) 285 (c) 19 (*d) 295
Directions (17) : In each of the following questions various terms of an alphabet series are given with one or more
terms missing as shown by (?). Choose the missing terms out of the given alternatives.
17. AZY,BUT, CXW, DWV, ?
(a) EVA (*b) EVU (c) VUE (d) VEU
18. What was the day on 31t October 19847
(a) Friday (b) Sunday (c*) Wednesday (d) Monday

19. The angle between the hands of clock when the time shown is 12 minutes past 5 o’clock is
(a*) 849 (b) 80° (c) 820 d) 760
20. A clock is set right at 5 am. If it loses 16 minutes in 24 hours, then what will be the true time when the clock
indicates 10 pm on the 3™ day?
(*fa) 11pm (b) 12pm (c) 1pm (d) 12.30 pm

Directions (21 to 22) : Study the following information carefully and answer the questions given below:
Eight friends Q, R, S, T, V, W,Y and Z are sitting around a circular table facing the center. There are three
males and five females in the group of friends. No two males are immediate neighbors of each other.
(i) V sits second to the right of his wife.
(ii) S sits third to the right of V.
(iii) W sits second to the right of her husband Z, Z is not an immediate neighbour of V's wife.
(iv) T is @a male and Y is not an immediate neighbor of V.
(v) R sits second to the right of Q.
21. Who amongst the following is V's wife?
(@ Q (b)Y () R d T
22. Which of the following pairs represents the immediate neighbours of T?
(@ RQ (b) Wz () YV (*d) WY
Directions (23 to 24) : In the figure, the circle stands for employed , the square stands for hard working, the triangle
stands for rural and the rectangle stands for intelligent. Study the figure carefully and answer the
question that follows

23. Non-rural, employed people who are neither intelligent nor hard working are represented by region
(@ 12 (b)) 11 (c) 10 ¢d)y 7

24. Rural people who are hard working and employed but not intelligent are indicated by region
(@ 1 (G () 3 d 4
25. What is my father’s son’s son to my son?
(@) Uncle (*b) Cousin Brother or Brother
(c) Son (d) none of these
26.
4f12|n]s
6|7]10]3
8flofw]7
7]5|7?|4a

(a) 12 (b) 14 (c) 13 (d*) 8


SHIKHAR
NT-JEE
27. What number of dots will be there on the face opposite to the face that contains 2 dots?
* A 0
. . ',
il .

@) 1 () 3 (c) 4 d 6
28. How many small cubes will have only one face coloured?
(@) 12 (b) 28
(¢ 22 d) 16
29. How many squares are there in the figure?
(@ 15 (b) 16
(e) 17 (d) 18

30. Find the number of triangles in the given figure.

(a) 20 (b) 22 (c) 16 (*d) 28


PART - III : SCIENCE

(Single Correct Choice Type)


This Section contains 15 Single choice questions. Each question has four choices (A), (B), (C) and (D) out
of which ONLY ONE is correct.
Marking Scheme:
You will be awarded 3 marks for correct answer, -1 for wrong answer and zero if Question is left un-
attempted.
31. A train passes over a 400 m long bridge. If the speed of the train is 30 m/s and the train takes 20 s to cross the
bridge, then the length of the train is
(a) 400m (b) 600 m (c) 800m (d*) 200m
32. A vehicle moving along a linear road takes the road it will go in 4t.
Speed

2V ey

B ]
;
i 1
|H | Ry
0 t 4t Time
Since the velocity-time graph of the vehicle is as shown in the figure, how many t took the vehicle to the first half
of the road?

@ 1 o) 22 © 2 @ 32
33. When an object undergoes acceleration
(a) lts speed always changes (*b) Its velocity always changes
(c) Iltalways falls towards the earth (d) Directing of motion always change

34. If both the mass and radius of the earth, each decrease by 50%, the acceleration due to gravity would
(a) increased by 50% (b) decrease by 50% (c*) increase by 100% (d) remain the same

35. A luminous object is placed 60 cm from surface of a convex mirror and a plane mirror is set so that virtual
images formed in two mirrors coincide. If plane mirror is at a distance of 40 cm from object, then focal length
of convex mirror is
(@ 25cm (*b) 30cm (c) 40cm (d) 60cm

36. What is the resistance between A and B in the following circuit (figure)?
10 10
10
s 19

a2 AV B
20
" 1 3
(fa) 1Q b) 20 (©) EQ (d) 50
NT-JEE
37. The wire having a green plastic covering is a
(@) live wire (b) neutral wire (c*) earth wire (d) none of these

38. The wire having a red plastic covering is a


(*a) live wire (b) neutral wire (c) earth wire (d) none of these

39. Which of the following has maximum number of atoms?


(@) 18gH0 (b) 18gO0: (*c) 18gCO2 (d) 18gCHa
40. The pH of the gastric juices released during digestion is
(*a) lessthan7 (b) more than7 (c) equalto7 (d) equalto0

41. Which of the following is incorrect about the ??CI’ ion?


(a) lts mass number is 35 (b) The number of protons is 17
(*¢) The number of electrons is 16 (d) The number of neutrons is 18

42. On moving from left to right across a period in the modern periodic table, the valency
(a) increase (b) decrease
(c*) firstincreases then decreases (d) first decreases then increases

43. Which of the following has a triple covalent bond?


(@) PHs (b) CO2 (c) ACk (@) N2
44. Which of the following is not a homologue of other three
(@) CHs (b) CoHs (c) CsHs (*d) CaHs
45. Anode mud refers to
(a) polarity of anode (b) metal of anode
(c*) impurities collected at bottom of anode in electrolysis during purification of metals
(d) Allof these

(Subjective Type Questions)

PART MATHS

This Section contains 9 Subjective Type Questions. Write solution for each question along with final
answer within the space provided.
Marking Scheme:
You will be awarded 5 marks for correct solution with correct answer and zero in all other cases. There is
no step marking.
46. Find the largest number that will divide 398, 436 and 542 leaving remainder 7, 11 and 15 respectively.
Solution: 17
47. In the given figure, in AABC, XY|| AC and XY divides the AABC into two regions such

that ar(ABXY) = 3 ar(ACYX). Determine%.

Solution: 1
2
48. If the points A( —4, -8), B(—4, 2), C(0, —5) and D(0, 5) are the vertices of parallelogram
ABCD, then taking AB as the base, find the height of the parallelogram.
Solution: 8
49. From the top of a building 60 m high, the angles of depression of the top and bottom of a vertical lamp post are
observed to be 30° and 60° respectively. Find
(i) The horizontal distance between the building and the lamp post.
(ii)The height of the lamp post. [ take «/5: 1.73]
Solution: (i) 203m (ii) 40m. »(‘
50. In figure, two equal circles, with centres O and O’ touch each other at X OO’ A A‘
produced meets the circles with centre O "at A, AC is tangent to the circle O
with centre O, at the point C, OD is perpendicular to AC. Find the value of
DO’
co’

Solution: 1
3

SHIKHAR : Indra Complex Vijaya Nagar, Chetakpuri Gw:


SHIKHAR
NT-JEE
51. Inthe figure If £ APB = 80° , find ZAOB, ZPAB and ZABP.

A
V=g D
B <\
A

Solution: 1009,

52. In the given figure, ABC is a right triangle right angled at A. If AB =6 cm , BC = 10 A


cm and O is the centre of the incricle of A ABC, the area of shaded region= (24— C B
km)em2. Find k.
Solution: 4
53. A cone of radius 10 cm is divided into two parts by a plane parallel to its base through the mid-point of its
height. Compare the volume of the two parts

Solution: 1
7
54. The probability of selecting a red ball at random from a jar that contains only red, blue and orange balls is Y.
The probability of selecting a blue ball at random from the same jar is 1/3. If the jar contains 15 orange balls,
find the total number of balls in the jar.
Solution: 36

PART - II : SCIENCE

(Subjective Type)
This Section contains 6 Subjective Type Questions. Write solution for each question along with final
answer within the space provided.
Marking Scheme:
You will be awarded 5 marks for correct solution with correct answer and zero in all other cases. There is
no step marking.
55. A particle is projected vertically upward from ground upward from ground level with a speed of 50 ms~!. For how
long will it be more than 105 m above the ground?
Solution: 4 seconds
56. Ray diagram of incident ray | is shown in the diagram of lens-mirror system of one double convex lens and
concave mirror
If all the dots are equally placed find the value of %
L

Principal
axis

L 3R
Solution: 1 A R
57. The three resistance A, B and C have values 3R,6Rand R respectively. — 6R VW —e
When some potential difference is applied across the network, the c
thermal powers dissipated by A, B and C are in what ratio? B
Solution: 4:2:3
58. The total numbers of moles of atoms present in 40 g of Calcium carbonate having molecular mass equal to 100
u?
Solution: 2
59. Count the number of single and double bonds in Propanone. Write its formula and structure and calculate its
molecular mass.
Solution: Do it yourself
60. Write the chemical formula of all the following compounds.
Plaster of Paris, Sulphurous acid, cupric oxide, ethyl ethanoate, butyric acid.
Solution: Do it yourself
NOTE : For a wide range of Mental Ability and Aptitude questions, refer to NTSE Previous Year Questions,
also known as PYQs, which contain 100 MAT questions in each paper. You may also attempt the PCM
section of NTSE question paper sets of the last 10 years for enhancing your concept application and
improve your speed.
SHIKHAR : Indra Complex Vijaya Nagar, Chetakpuri Gw:

You might also like